Test #2 NURS 327 Prep-U

Pataasin ang iyong marka sa homework at exams ngayon gamit ang Quizwiz!

How long must apnea during sleep be in order to be considered OSA?

10 seconds or longer

The goal for oxygen therapy in COPD is to support tissue oxygenation, decrease the work of the cardiopulmonary system, and maintain the resting partial arterial pressure of oxygen (PaO2) of at least ______ mm Hg and an arterial oxygen saturation (SaO2) of at least ___%. 54 mm Hg; 84% 56 mm Hg; 86% 58 mm Hg; 88% 60 mm Hg; 90%

60 mm Hg; 90%

A patient is to receive an oxygen concentration of 70%. What is the best way for the nurse to deliver this concentration? A nasal cannula An oropharyngeal catheter A partial rebreathing mask A Venturi mask

A partial rebreathing mask Explanation: Partial rebreathing masks have a reservoir bag that must remain inflated during both inspiration and expiration. The nurse adjusts the oxygen flow to ensure that the bag does not collapse during inhalation. A high concentration of oxygen (50% to 75%) can be delivered because both the mask and the bag serve as reservoirs for oxygen. The other devices listed cannot deliver oxygen at such a high concentration.

Which assessment finding would be most consistent with advanced emphysema? Dependent edema Epigastric pain Barrel-shaped chest Aortic bruit

Barrel-shaped chest Explanation: Barrel chest occurs as result of overinflation of the lungs. In a client with emphysema, the ribs are more widely spaced and the intercostal spaces tend to bulge on expiration. The apprearance of a such a client with advanced emphysema is easily detected.

A client arrives in the emergency department reporting shortness of breath. She has 3+ pitting edema below the knees, a respiratory rate of 36 breaths per minute, and heaving respirations. The nurse auscultates the client's lungs to reveal coarse, moist, high-pitched, and non-continuous sounds that do not clear with coughing. The nurse will document these sounds as which type? Wheezes Rhonchi Crackles Pleural rub

Crackles Explanation: Crackles are adventitious breath sounds that are high-pitched, discontinuous, and popping; they may or may not clear with coughing and are moist. Often crackles are associated with heart failure.

A client diagnosed with acute respiratory distress syndrome (ARDS) is restless and has a low oxygen saturation level. If the client's condition does not improve and the oxygen saturation level continues to decrease, what procedure will the nurse expect to assist with in order to help the client breathe more easily? Intubate the client and control breathing with mechanical ventilation Increase oxygen administration Administer a large dose of furosemide (Lasix) IVP stat Schedule the client for pulmonary surgery

Intubate the client and control breathing with mechanical ventilation Explanation: A client with ARDS may need mechanical ventilation to assist with breathing while the underlying cause of the pulmonary edema is corrected. The other options are not appropriate.

The nurse is reviewing the blood gas results for a patient with pneumonia. What arterial blood gas measurement best reflects the adequacy of alveolar ventilation? PaO2 PaCO2 pH SaO2

PaCO2 Explanation: When the minute ventilation falls, alveolar ventilation in the lungs also decreases, and the PaCO2 increases.

A client with COPD has been receiving oxygen therapy for an extended period. What symptoms would be indicators that the client is experiencing oxygen toxicity? Select all that apply. Substernal pain Dyspnea Fatigue Mood swings Bradycardia

Substernal pain Dyspnea Fatigue

A client with chronic bronchitis is admitted with an exacerbation of symptoms. During the nursing assessment, the nurse will expect which of the following findings? Select all that apply. Use of accessory muscles to breathe Tympany percussed bilaterally over the lung bases Hypoventilatory breathing pattern Respiratory rate of 10 breaths per minute Purulent sputum with frequent coughing

Use of accessory muscles to breathe Purulent sputum with frequent coughing Hypoventilatory breathing pattern Correct response: Use of accessory muscles to breathe Purulent sputum with frequent coughing

Which of the following are indicators that a client is ready to be weaned from a ventilator? Select all that apply. Vital capacity of 13 mL/kg Tidal volume of 8.5 mL/kg Rapid/shallow breathing index of 112 breaths/min PaO2 of 64 mm Hg FiO2 45%

Vital capacity of 13 mL/kg Tidal volume of 8.5 mL/kg PaO2 of 64 mm Hg Explanation: Weaning criteria for clients are as follows: Vital capacity 10 to 15 mL/kg; Maximum inspiratory pressure at least -20 cm H2; Tidal volume: 7 to 9 mL/kg; Minute ventilation: 6 L/min; Rapid/shallow breathing index below 100 breaths/min; PaO2 > 60 mm Hg; FiO2 < 40%

A first-line antibiotic used to treat acute rhinosinusitis is ampicillin. amoxicillin-clavulanic acid. cefprozil. cefuroxime.

ampicillin. Explanation: First-line antibiotics include amoxicillin, ampicillin, and erythromycin. Second-line therapy includes cefuroxime, cefprozil, and amoxicillin-clavulanic acid.

What is pectus carinatum?

flattening of the thorax on either side with forward projection of the sternum resembling the keel of a boat **pidegon chest**

What is medicamentosa?

rebound congestion from decongestants

What is histologic hypoxia?

toxic substances (cyanide)

A client with pulmonary hypertension has a positive vasoreactivity test. What medication does the nurse anticipate administering to this client? Calcium channel blockers Angiotensin converting enzyme inhibitor Beta blockers Angiotensin receptor blockers

Calcium channel blockers Explanation: Clients with a positive vasoreactivity test may be prescribed calcium channel blockers. Calcium channel blockers have a significant advantage over other medications taken to treat PH in that they may be taken orally and are generally less costly; however, because calcium channel blockers are indicated in only a small percentage of clients, other treatment options, including prostanoids, are often necessary (Hopkins & Rubin, 2016).

Arterial blood gas analysis would reveal which value related to acute respiratory failure? PaO2 80 mm Hg pH 7.28 PaCO2 32 mm Hg pH 7.35

pH 7.28 Explanation: Acute respiratory failure is defined as a decrease in arterial oxygen tension (PaO2) to less than 60 mm Hg (hypoxemia) and an increase in arterial carbon dioxide tension (PaCO2) to greater than 50 mm Hg (hypercapnia), with an arterial pH less than 7.35.

What is anemia hypoxia?

ineffective Hgb **CO poisoning

A client admitted with pneumonia has a history of lung cancer and heart failure. A nurse caring for this client recognizes that he should maintain adequate fluid intake to keep secretions thin for ease in expectoration. The amount of fluid intake this client should maintain is: 1.4 L. 2 L. 3 L. unspecified.

1.4 L. Explanation: Clients need to keep their secretions thin by drinking 2 to 3 L of clear liquids per day. In clients with heart failure, fluid intake shouldn't exceed 1.5 L daily. Reference:

The herpes simplex virus type 1 (HSV-1), which produces a cold sore (fever blister), has an incubation period of 2 to 12 days. 20 to 30 days. 1 to 3 months. 3 to 6 months.

2 to 12 days. Explanation: HSV-1 is transmitted primarily by direct contact with infected secretions. The time periods of 20 to 30 days, 1 to 3 months, and 3 to 6 months exceed the incubation period.

Normally, approximately what percentage of the blood pumped by the right ventricle does not perfuse the alveolar capillaries? 2% 4% 6% 8%

2% Explanation: Normally, about 2% of the blood pumped by the right ventricle does not perfuse the alveolar capillaries. This shunted blood drains into the left side of the heart without participating in alveolar gas exchange. The other options are inaccurate.

The nurse is caring for a client in the ICU who required emergent endotracheal (ET) intubation with mechanical ventilation. The nurse receives an order to obtain arterial blood gases (ABGs) after the procedure. The nurse recognizes that ABGs should be obtained how long after mechanical ventilation is initiated? 10 minutes 15 minutes 20 minutes 25 minutes

20 minutes Explanation: The nurse records minute volume and obtains ABGs to measure carbon dioxide partial pressure (PaCO2), pH, and PaO2 after 20 minutes of continuous mechanical ventilation.

A 73-year-old client is admitted to the pulmonology unit of the hospital. She was admitted with a pleural effusion and was "tapped" to drain the fluid to reduce her mediastinal pressure. How much fluid is typically present between the pleurae, which surround the lungs, to prevent friction rub? 5-10 mL 20-25 mL 20-30 mL No fluid normally is present

20-25 mL Explanation: The pleural space, located between the visceral and parietal pleura, normally contains 20 mL of fluid or less. The fluid helps lubricate the visceral and parietal pleura.

A client with emphysema informs the nurse, "The surgeon will be removing about 30% of my lung so that I will not be so short of breath and will have an improved quality of life." What surgery does the nurse understand the surgeon will perform? A sleeve resection A lung volume reduction A wedge resection Lobectomy

A lung volume reduction

What is circulatory hypoxia?

inadequate capillary circulation

What is eupnea?

normal, unlabored breathing

What is tactile fremitus associated with? What is it?

pneumonia vibration when patient is saying constant sounds

What is hypoexmic hypoxia?

decresed 02 level in blood

After diagnosing a client with pulmonary tuberculosis, the physician tells family members that they must receive isoniazid (INH [Laniazid]) as prophylaxis against tuberculosis. The client's daughter asks the nurse how long the drug must be taken. What is the usual duration of prophylactic isoniazid therapy? 3 to 5 days 1 to 3 weeks 2 to 4 months 6 to 12 months

6 to 12 months Explanation: Prophylactic isoniazid therapy must continue for 6 to 12 months at a daily dosage of 300 mg. Taking the drug for less than 6 months may not provide adequate protection against tuberculosis.

A client with exacerbation of chronic obstructive pulmonary disease (COPD) is scheduled for a thoracentesis. Which nursing intervention would be appropriate for client saftey? Administering a prn cough suppressant Assisting the client to a prone position Obtaining arterial blood gas values immediately after the procedure Applying oxygen via nasal cannula

Administering a prn cough suppressant Explanation: A thoracentesis is performed to remove fluid and air from the pleural cavity and to aspirate pleural fluid for analysis, pleural biospy, or instillation of medication into the pleural space. The client is placed in an upright position with adequate supports for comfort. The nurse will encourage the client to refrain from coughing. Any sudden and unexpected movement, such as coughing, can traumatize the visceral pleura and lung. Clients experiencing exacerbation of COPD often have a productive cough; therefore, administration of a prn cough medication would be helpful to suppress any potential coughing during the procedure.

A client is admitted to the emergency department with a stab wound and is now presenting dyspnea, tachypnea, and sucking noise heard on inspiration and expiration. The nurse should care for the wound in which manner? Clean the wound and leave open to the air. Apply vented dressing. Apply airtight dressing. Apply direct pressure to the wound.

Apply airtight dressing. Explanation: The client has developed a pneumothorax, and the best action is to prevent further deflation of the affected lung by placing an airtight dressing over the wound. A vented dressing would be used in a tension pneumothorax, but because air is heard moving in and out, a tension pneumothorax is not indicated. Applying direct pressure is required if active bleeding is noted.

Which ventilator mode provides full ventilatory support by delivering a present tidal volume and respiratory rate? IMV SIMV Assist control Pressure support

Assist control Explanation: Assist-control ventilation provides full ventilator support by delivering a preset tidal volume and respiratory rate. IMV provides a combination of mechanically assisted breaths and spontaneous breaths. SIMV delivers a preset tidal volume and number of breaths per minute. Between ventilator-delivered breaths, the patient can breathe spontaneously with no assistance from the ventilator for those extra breaths. Pressure support ventilation assists SIMV by applying a pressure plateau to the airway throughout the client-triggered inspiration to decrease resistance within the tracheal tube and ventilator tubing.

The nurse should be alert for a complication of bronchiectasis that results from a combination of retained secretions and obstruction and that leads to the collapse of alveoli. What complication should the nurse monitor for? Atelectasis Emphysema Pleurisy Pneumonia

Atelectasis

A thoracentesis is performed to obtain a sample of pleural fluid or a biopsy specimen from the pleural wall for diagnostic purposes. What does serous fluid indicate? Trauma Infection Cancer Emphysema

Cancer Explanation: A thoracentesis may be performed to obtain a sample of pleural fluid or to biopsy a specimen from the pleural wall for diagnostic purposes. Serous fluid may be associated with cancer, inflammatory conditions, or heart failure. Blood fluid typically suggests trauma. Purulent fluid is diagnostic for infection. Complications that may follow a thoracentesis include pneumothorax and subcutaneous emphysema.

Which drug is second-line pharmacotherapy for smoking abstinence? Nicotine gum Clonidine Nortriptyline Buproprion SR

Clonidine Explanation: Second-line pharmacotherapy includes the antihypertensive agent clonidine. However, its use is limited by its side effects. First-line therapy includes nicotine gum, nortriptyline, and buproprion SR. Reference:``

The nurse is auscultating the patient's lung sounds to determine the presence of pulmonary edema. What adventitious lung sounds are significant for pulmonary edema? Crackles in the lung bases Low-pitched rhonchi during expiration Pleural friction rub Sibilant wheezes

Crackles in the lung bases Explanation: When clinically significant atelectasis develops, it is generally characterized by increased work of breathing and hypoxemia. Decreased breath sounds and crackles are heard over the affected area.

A patient is prescribed a mast cell stabilizer for the treatment of asthma. Which commonly used medication will the nurse educate the patient about? Albuterol Budesonide Cromolyn sodium Theophylline

Cromolyn sodium Explanation: Cromolyn sodium (Crolom, NasalCrom) and nedocromil (Alocril, Tilade) are mild to moderate anti-inflammatory agents and are considered alternative medications for treatment. These medications stabilize mast cells. The other medications listed are not mast cell stabilizers.

A client has asthma. Which of the following medications is a commonly prescribed mast cell stabilizer used for asthma? Albuterol Budesonide Cromolyn sodium Theophylline

Cromolyn sodium Explanation: Cromolyn sodium and nedocromil are mild to moderate anti-inflammatory agents and are considered alternative medications for treatment. These medications stabilize mast cells. Albuterol is a long-acting beta2-antagonist. Budesonide is an inhaled corticosteroid. Theophylline is a mild to moderate bronchodilator.

Which medication is contraindicated in acute asthma exacerbations? Albuterol Cromolyn sodium Levalbuterol Ipratropium bromide

Cromolyn sodium Explanation: Cromolyn sodium is contraindicated in clients with acute asthma exacerbation.

When the nurse is assessing the older adult patient, what gerontologic changes in the respiratory system should the nurse be aware of? (Select all that apply.) Decreased alveolar duct diameter Increased presence of mucus Decreased gag reflex Increased presence of collagen in alveolar walls Decreased presence of mucus

Decreased gag reflex Increased presence of collagen in alveolar walls Decreased presence of mucus

The nurse is using an in-line suction kit to suction a patient who is intubated and on a mechanical ventilator. What benefits does inline suction have for the patient? (Select all that apply.) Decreases hypoxemia Decreases patient anxiety Sustains positive end expiratory pressure (PEEP) Increases oxygen consumption Prevents aspiration

Decreases hypoxemia Decreases patient anxiety Sustains positive end expiratory pressure (PEEP) Explanation: An in-line suction device allows the patient to be suctioned without being disconnected from the ventilator circuit. In-line suctioning (also called closed suctioning) decreases hypoxemia, sustains PEEP, and can decrease patient anxiety associated with suctioning (Sole et al., 2013).

A patient is admitted to the hospital with pulmonary arterial hypertension. What assessment finding by the nurse is a significant finding for this patient? Ascites Dyspnea Hypertension Syncope

Dyspnea Explanation: Dyspnea, the main symptom of PH, occurs at first with exertion and eventually at rest. Substernal chest pain also is common. Other signs and symptoms include weakness, fatigue, syncope, occasional hemoptysis, and signs of rightsided heart failure (peripheral edema, ascites, distended neck veins, liver engorgement, crackles, heart murmur). Anorexia and abdominal pain in the right upper quadrant may also occur.

During a preadmission assessment, for what diagnosis would the nurse expect to find decreased tactile fremitus and hyperresonant percussion sounds? Bronchitis Emphysema Atelectasis Pulmonary edema

Emphysema Explanation: Emphysema is associated with decreased tactile fremitus and hyperresonant percussion sounds. Bronchitis is associated with normal tactile fremitus and resonant percussion sounds. Atelectasis is associated with absent tactile fremitus and dull percussion sounds. Pulmonary edema is associated with normal tactile fremitus and resonant percussion sounds.

Which of the following is a common irritant that acts as a trigger of asthma? Esophageal reflux Peanuts Aspirin sensitivity Molds

Esophageal reflux Explanation: Esophageal reflux, viral respiratory infections, cigarette smoke, and exercise are all irritants that can trigger asthma. Peanuts, aspirin sensitivity, and molds are antigens

A patient with sinus congestion points to a location on the inside of his eye as the area of pain. The nurse documents that the patient is complaining of pain in which sinus? Frontal Ethmoid Maxillary Sphenoid

Ethmoid Explanation: The ethmoidal sinuses are located between the eyes and behind the nose. Inflammation and swelling block drainage into the nose; eventually an infection results.

A client is receiving mechanical ventilation. How frequently should the nurse auscultate the client's lungs to check for secretions? Every 30 to 60 minutes Every 1 to 2 hours Every 2 to 4 hours Every 4 to 6 hours

Every 2 to 4 hours Explanation: Continuous positive-pressure ventilation increases the production of secretions regardless of the patient's underlying condition. The nurse assesses for the presence of secretions by lung auscultation at least every 2 to 4 hours.

The nursing instructor is talking with the junior class of nursing students about lung cancer. What would be the best rationale the instructor could give for the difficulty of early diagnosis of lung cancer? Symptoms are often minimized by clients. There are no early symptoms of lung cancer. Symptoms often mimic other infectious diseases. Symptoms often do not appear until the disease is well established.

Explanation: Early diagnosis of cancer of the lung is difficult because symptoms often do not appear until the disease is well established. Option A is correct, but it is not the best answer. Option B is incorrect because it is not a true statement. Option C is incorrect because lung cancer is not an infectious disease.

A 76-year-old client presents to the ED reporting "laryngitis." The triage nurse should ask whether the client has a medical history that includes Gastroesophageal reflux disease (GERD) Chronic obstructive pulmonary disease (COPD) Congestive heart failure (CHF) Respiratory failure (RF)

Gastroesophageal reflux disease (GERD) Explanation: The nurse should ask whether the client has a medical history of GERD. Laryngitis is common in older adults and may be secondary to GERD. Older adults are more likely to have impaired esophageal peristalsis and a weaker esophageal sphincter. COPD, CHF, and RF are not associated with laryngitis in the older adult.

The nurse should monitor a client receiving mechanical ventilation for which of the following complications? Gastrointestinal hemorrhage Immunosuppression Increased cardiac output Pulmonary emboli

Gastrointestinal hemorrhage Explanation: Gastrointestinal hemorrhage occurs in approximately 25% of clients receiving prolonged mechanical ventilation. Other possible complications include incorrect ventilation, oxygen toxicity, fluid imbalance, decreased cardiac output, pneumothorax, infection, and atelectasis. Immunosuppression and pulmonary emboli are not direct consequences of mechanical ventilation.

What are the parameters for the grades of COPD?

Grade I- Mild FEV1/FVC <70%, FEV1 ≥80% predicted Grade II- Moderate FEV1/FVC <70%, FEV1 50-80% predicted Grade III- Severe FEV1/FVC <70%, FEV1 <30-50% predicted Grade IV- Very severe FEV1/FVC <70%, FEV1 <30% predicted

The nurse is educating the patient in the use of a mini-nebulizer. What should the nurse encourage the patient to do? (Select all that apply.) Hold the breath at the end of inspiration for a few seconds. Cough frequently. Take rapid, deep breaths. Frequently evaluate progress. Prolong the expiratory phase after using the nebulizer.

Hold the breath at the end of inspiration for a few seconds. Cough frequently. Frequently evaluate progress. Explanation: The nurse instructs the patient to breathe through the mouth, taking slow, deep breaths, and then to hold the breath for a few seconds at the end of inspiration to increase intrapleural pressure and reopen collapsed alveoli, thereby increasing functional residual capacity. The nurse encourages the patient to cough and to monitor the effectiveness of the therapy. The nurse instructs the patient and family about the purpose of the treatment, equipment setup, medication additive, and proper cleaning and storage of the equipment.

Which type of oxygen therapy includes the administration of oxygen at pressure greater than atmospheric pressure? Low-flow systems Hyperbaric High-flow systems Transtracheal

Hyperbaric Explanation: Hyperbaric oxygen therapy is the administration of oxygen at pressures greater than atmospheric pressure. As a result, the amount of oxygen dissolved in plasma is increased, which increases oxygen levels in the tissues. Low-flow systems contribute partially to the inspired gas the client breathes, which means that the client breathes some room air along with the oxygen. High-flow systems are indicated for clients who require a constant and precise amount of oxygen. During transtracheal oxygenation, clients achieve adequate oxygenation at lower rates, making this method less expensive and more efficient.

Which clinical manifestation of hemorrhage is related to carotid artery rupture? Increased pulse rate Increased blood pressure Shallow respirations Dry skin

Increased pulse rate Explanation: The nurse monitors vital signs for changes, particularly increased pulse rate, decreased blood pressure, and rapid, deep respirations. Cold, clammy, pale skin may indicate active bleeding.

A client continues to report dyspnea on exertion and overall weakness. A pulmonary artery catheter is placed and the mean pulmonary arterial pressure is 35 mm Hg. What condition is the client experiencing? Pulmonary arterial hypertension Restrictive lung disease Asthma Atelectasis

Pulmonary arterial hypertension Explanation: Pulmonary hypertension (PH) is confirmed with a mean pulmonary artery pressure greater than 25 mm Hg. The main symptom of PH is dyspnea. Client with restrictive lung disease, asthma, and atelectasis should not have PH.

A nursing student understands that emphysema is directly related to which of the following? Diminished alveolar surface area Hypercapnia resulting from decreased carbon dioxide tension Hypoxemia secondary to impaired oxygen diffusion Respiratory acidosis from airway obstruction

Respiratory acidosis from airway obstruction Explanation: In the later stages of emphysema, carbon dioxide elimination is impaired, resulting in increased carbon dioxide tension in arterial blood (hypercapnia) leading to respiratory acidosis.

Which ventilation-perfusion ratio is exhibited by acute respiratory distress syndrome (ARDS)? Silent unit Dead space Low ventilation-perfusion ratio Normal ratio of perfusion to ventilation

Silent unit Explanation: When ventilation exceeds perfusion a dead space exists. An example of a dead space is a pulmonary emboli. A low ventilation-perfusion ratio exists in pneumonia or with a mucus plug. A silent unit occurs in pneumothorax or ARDS.

A client with myasthenia gravis is receiving continuous mechanical ventilation. When the high-pressure alarm on the ventilator sounds, what should the nurse do? Check for an apical pulse. Suction the client's artificial airway. Increase the oxygen percentage. Ventilate the client with a handheld mechanical ventilator.

Suction the client's artificial airway.

Perfusion refers to blood supply to the lungs, through which the lungs receive nutrients and oxygen. What are the two methods of perfusion? The two methods of perfusion are the bronchial and alveolar circulation. The two methods of perfusion are the bronchial and capillary circulation. The two methods of perfusion are the bronchial and pulmonary circulation. The two methods of perfusion are the alveolar and pulmonary circulation.

The two methods of perfusion are the bronchial and pulmonary circulation. Explanation: The two methods of perfusion are the bronchial and pulmonary circulation. There is no alveolar circulation. Capillaries are the vessels that performs the perfusion regardless of which area of the lung they are in.

A client finished a course of antibiotics for laryngitis but continues to experience persistent hoarseness. Which symptom would cause the nurse to suspect laryngeal cancer? a feeling of swelling at the back of the throat weight loss discomfort when drinking cold liquids headaches in the morning

a feeling of swelling at the back of the throat Explanation: After an initial hoarseness lasting longer than a month, clients with laryngeal cancer will feel a sensation of swelling or a lump in the throat or in the neck. Weight loss often occurs later in the progression of laryngeal cancer due to reduced calorie intake as a result of impaired swallowing and pain. Clients with laryngeal cancer may report burning in the throat when swallowing hot or citrus liquids. Clients with obstructive sleep apnea may experience a morning headache.

A client arrives in the emergency department reporting shortness of breath. She has 3+ pitting edema below the knees, a respiratory rate of 36 breaths per minute, and heaving respirations. The nurse auscultates the client's lungs to reveal coarse, moist, high-pitched, and non-continuous sounds that do not clear with coughing. The nurse will document these sounds as which type? Wheezes Rhonchi Crackles Pleural rub

crackles

A client who is diagnosed with chronic respiratory failure will have which symptom? dyspnea hypoxemia hypercapnia ventilatory failure

dyspnea Explanation: Apprehension, restlessness, fatigue, headache, dyspnea, wheezing, cyanosis, and use of the accessory muscles of respiration are seen in clients with impending respiratory failure. A fall in arterial oxygen levels, or hypoxemia, is a sign of acute respiratory failure. A rise in arterial CO2, or hypercapnia, is a sign of acute respiratory failure. Ventilatory failure develops in acute respiratory failure when the alveoli cannot adequately expand. Reference:

A nurse is teaching a client about using an incentive spirometer. Which statement by the nurse is correct? "Breathe in and out quickly." "You need to start using the incentive spirometer 2 days after surgery." "Before you do the exercise, I'll give you pain medication if you need it." "Don't use the incentive spirometer more than 5 times every hour."

"Before you do the exercise, I'll give you pain medication if you need it." Explanation: The nurse should assess the client's pain level before the client does incentive spirometry exercises and administer pain medication as needed. Doing so helps the client take deeper breaths and help prevents atelectasis. The client should breathe in slowly and steadily, and hold his breath for 3 seconds after inhalation. The client should start doing incentive spirometry immediately after surgery and aim to do 10 incentive spirometry breaths every hour.

A nurse has just completed teaching with a client who has been prescribed a meter-dosed inhaler for the first time. Which statement if made by the client would indicate to the nurse that further teaching and follow-up care is necessary? "I will make sure to take a slow, deep breath as I push on my inhaler." "After I breathe in, I will hold my breath for 10 seconds." "I do not need to rinse my mouth with this type of inhaler." "If I use the spacer, I know I am only supposed to push on the inhaler once."

"I do not need to rinse my mouth with this type of inhaler." Explanation: Mouth-washing and spitting are effective in reducing the amount of drug swallowed and absorbed systemically. Actuation during a slow (30 L/min or 3 to 5 seconds) and deep inhalation should be followed by 10 seconds of holding the breath. The client should actuate only once. Simple tubes do not obviate the spacer/VHC per inhalation.

A nurse is caring for a client who has a history of sleep apnea. The client understands the disease process when he says: "I need to keep my inhaler at the bedside." "I should eat a high-protein diet." "I should become involved in a weight loss program." "I should sleep on my side all night long."

"I should become involved in a weight loss program." Explanation: Obesity and decreased pharyngeal muscle tone commonly contribute to sleep apnea; the client may need to become involved in a weight loss program. Using an inhaler won't alleviate sleep apnea, and the physician probably wouldn't order an inhaler unless the client had other respiratory complications. A high-protein diet and sleeping on the side aren't treatment factors associated with sleep apnea. Reference:

The nurse is caring for a client with an exacerbation of COPD and scheduled for pulmonary function studies using a spirometer. Which client statement would the nurse clarify? "My study is scheduled for 10 AM, several hours after I eat." "I brought comfortable clothes and shoes for the test." "I am ordered a bronchodilator to note lung improvement following use." "I will breathe in through my mouth and out through my nose."

"I will breathe in through my mouth and out through my nose." Explanation: The nurse would clarify the client's statement of improper breathing technique. During a pulmonary function test using a spirometer, a nose clip prevents air from escaping through the client's nose when blowing into the spirometer. All other statements are correct.

A client admitted to the facility for treatment for tuberculosis receives instructions about the disease. Which statement made by the client indicates the need for further instruction? "I'll have to take the medication for up to a year." "This disease may come back later if I am under stress." "I'll stay in isolation for 6 weeks." "I'll always have a positive test for tuberculosis."

"I'll stay in isolation for 6 weeks." Explanation: The client requires additional teaching if he states that he'll be in isolation for 6 weeks. The client needs to be in isolation for 2 weeks, not 6, while taking the tuberculosis drugs. After 2 weeks of antitubercular therapy, the client is no longer considered contagious. The client needs to receive the drugs for 9 months to a year. He'll be positive when tested and if he's sick or under some stress he could have a relapse of the disease.

A client is being evaluated for possible lung cancer. Which client statement most likely indicates lung cancer? "My cough has changed from a dry cough to one with lots of sputum production." "I've had a low-grade fever for 2 weeks." "My voice is hoarser than it used to be." "I've lost 10 pounds in the last month."

"My cough has changed from a dry cough to one with lots of sputum production." Explanation: A cough that changes in character is one of the hallmark signs of lung cancer. Low-grade fever, hoarseness, and weight loss may be attributed to other disease processes and don't necessarily indicate lung cancer. Reference:

A nurse recognizes that a client with tuberculosis needs further teaching when the client states: "I'll have to take these medications for 9 to 12 months." "It won't be necessary for the people I work with to take medication." "I'll need to have scheduled laboratory tests while I'm on the medication." "The people I have contact with at work should be checked regularly."

"The people I have contact with at work should be checked regularly." Explanation: The client requires additional teaching if he states that coworkers need to be checked regularly. Such casual contacts needn't be tested for tuberculosis. However, a person in close contact with a person who's infectious is at risk and should be checked. The client demonstrates effective teaching if he states that he'll take his medications for 9 to 12 months, that coworkers don't need medication, and that he requires laboratory tests while on medication. Coworkers not needing medications, taking the medication for 9 to 12 months, and having scheduled laboratory tests are all appropriate statements.

An obese male is being evaluated for OSA. The nurse asks the patient's wife to document the number and frequency of incidences of apnea while her husband is asleep. The nurse tells the wife that a characteristic indicator of OSA is a breathing cycle characterized by periods of breathing cessation for: 4 seconds with 2 episodes/hour. 6 seconds with 3 episodes/hour. 8 seconds with 4 episodes/hour. 10 seconds with 5 episodes/hour.

10 seconds with 5 episodes/hour. Explanation: OSA is characterized by frequent and loud snoring, with breathing cessation for 10 seconds or longer, for at least five episodes per hour, followed by abrupt awakening with a loud snort as the blood oxygen level drops. Symptoms typically progress with weight gain, aging, and during the transition to menopause for women.

The nurse is administering anticoagulant therapy with heparin. What International Normalized Ratio (INR) would the nurse know is within therapeutic range? 0.5 to 1.0 1.5 to 2.5 2.0 to 2.5 3.0 to 3.5

2.0 to 2.5

What should an ET cuff pressure be?

20-24mmHg

A client has a tracheostomy but doesn't require continuous mechanical ventilation. When weaning the client from the tracheostomy tube, the nurse initially should plug the opening in the tube for: 15 to 60 seconds. 5 to 20 minutes. 30 to 40 minutes. 45 to 60 minutes.

5 to 20 minutes. Explanation: Initially, the nurse should plug the opening in the tracheostomy tube for 5 to 20 minutes, then gradually lengthen this interval according to the client's respiratory status. A client who doesn't require continuous mechanical ventilation already is breathing without assistance, at least for short periods; therefore, plugging the opening of the tube for only 15 to 60 seconds wouldn't be long enough to reveal the client's true tolerance to the procedure. Plugging the opening for more than 20 minutes would increase the risk of acute respiratory distress because the client requires an adjustment period to start breathing normally.

A client who has started therapy for drug-resistant tuberculosis demonstrates understanding of tuberculosis transmission when he says: "My tuberculosis isn't contagious after I take the medication for 24 hours." "I'm clear when my chest X-ray is negative." "I'm contagious as long as I have night sweats." "I'll stop being contagious when I have a negative acid-fast bacilli test."

A client with drug-resistant tuberculosis isn't contagious when he's had a negative acid-fast test. A client with nonresistant tuberculosis is no longer considered contagious when he shows clinical evidence of decreased infection, such as significantly decreased coughing and fewer organisms on sputum smears. The medication may not produce negative acid-fast test results for several days. The client won't have a clear chest X-ray for several months after starting treatment. Night sweats are a sign of tuberculosis, but they don't indicate whether the client is contagious.

Which ventilation-perfusion ratio is exhibited by a pulmonary embolus? Low ventilation-perfusion ratio Dead space Silent unit Normal ratio of perfusion to ventilation

A dead space exists when ventilation exceeds perfusion. An example of a dead space is a pulmonary embolus. A low ventilation-perfusion ratio exists in pneumonia or with a mucus plug. A silent unit occurs in pneumothorax or acute respiratory distress syndrome.

What is pectus excavatum?

A markedly sunken sternum and adjacent cartilages (also called a funnel breast). Depression begins at second intercostal space, becoming depressed most at junction of xiphoid with body of sternum. More noticable on inspiration. Congenital, usually not symptomatic. When severe, sternal depression may cause embarrassment and a negative self-concept. Surgery may be indicated.

A nurse is instructing a client who is scheduled for a perfusion lung scan. What teaching should the nurse include in the information about the procedure? Select all that apply. A mask will be placed over the nose and mouth during the test. The client will be expected to lie under the camera. The imaging time will amount to 20 to 40 minutes. The client will be expected to be NPO for 12 hours prior to the procedure. An injection will be placed into the lung during the procedure.

A mask will be placed over the nose and mouth during the test. The client will be expected to lie under the camera. The imaging time will amount to 20 to 40 minutes. An injection will be placed into the lung during the procedure. Explanation: A ventilation/perfusion lung scan is performed by injecting a radioactive agent into a peripheral vein and then obtaining a scan of the chest to detect radiation. The isotope particles pass through the right side of the heart and are distributed into the lungs in proportion to the regional blood flow, making it possible to trace and measure blood perfusion through the lung. This procedure is used clinically to measure the integrity of the pulmonary vessels relative to blood flow and to evaluate blood flow abnormalities, as seen in pulmonary emboli. The imaging time is 20 to 40 minutes, during which the client lies under the camera with a mask fitted over the nose and mouth. Imaging is followed by the ventilation component of the scan. The client need not be NPO for 12 hours prior to the procedure.

A surgeon completes a total laryngectomy. Postoperatively, the nurse explains to the patient's family that: One vocal cord was removed along with a portion of the larynx. The voice was spared and a tracheostomy would be in place until the airway was established. A permanent tracheal stoma would be necessary. A portion of the vocal cord was removed.

A permanent tracheal stoma would be necessary. Explanation: A total laryngectomy will result in a permanent stoma and total loss of voice. A partial laryngectomy involves the removal of one vocal cord. The voice is spared with the supraglottic laryngectomy. Removal of a portion of the vocal cord occurs with a hemilaryngectomy.

A client with thoracic trauma is admitted to the ICU. The nurse notes the client's chest and neck are swollen and there is a crackling sensation when palpated. The nurse consequently identifies the presence of subcutaneous emphysema. If this condition becomes severe and threatens airway patency, what intervention is indicated? A chest tube A tracheostomy An endotracheal tube A feeding tube

A tracheostomy

Which type of lung cancer is the most prevalent among both men and women? Large cell carcinoma Squamous cell carcinoma Adenocarcinoma Small cell carcinoma

Adenocarcinoma Explanation: Adenocarcinoma is most prevalent in both men and women and presents more peripherally as masses or nodules and often metastasizes. Large cell carcinoma is a fast-growing tumor that tends to arise peripherally. Squamous cell carcinoma is more centrally located and arises more commonly in the segmental and subsegmental bronchi in response to repetitive carcinogenic exposure. Small cell carcinomas arise primarily as proximal lesions, but may arise in any part of the tracheobronchial tree.

The nurse is caring for a client who has recurrent sinusitis. Which consideration could the nurse suggest to best decrease the frequency of infections? Administer an over-the-counter decongestant. Use an anti-allergy medication to decrease rhinitis. Place a warm cloth over the sinus area of the forehead. Gently blow the nose to eliminate nasal secretions.

Administer an over-the-counter decongestant. Explanation: The principle causes of sinusitis are the spread of infection from the nasal passages to the sinus and the blockage of normal sinus drainage. Interference with sinus drainage predisposes a client to sinusitis. Administering a decongestant opens the nasal passages for drainage. The other options can be helpful for a sinus infection, but opening the passages is best.

A physician orders triamcinolone and salmeterol for a client with a history of asthma. What action should the nurse take when administering these drugs? Administer the triamcinolone and then administer the salmeterol. Administer the salmeterol and then administer the triamcinolone. Allow the client to choose the order in which the drugs are administered. Monitor the client's theophylline level before administering the medications.

Administer the salmeterol and then administer the triamcinolone. Explanation: A client with asthma typically takes bronchodilators and uses corticosteroid inhalers to prevent acute episodes. Triamcinolone (Azmacort) is a corticosteroid; Salmeterol (Serevent) is an adrenergic stimulant (bronchodilator). If the client is ordered a bronchodilator and another inhaled medication, the bronchodilator should be administered first to dilate the airways and to enhance the effectiveness of the second medication. The client may not choose the order in which these drugs are administered because they must be administered in a particular order. Monitoring the client's theophylline level isn't necessary before administering these drugs because neither drug contains theophylline.

A client is being treated in the ED for respiratory distress coupled with pneumonia. The client has no medical history. However, the client works in a coal mine and smokes 10 cigarettes a day. The nurse anticipates which order based on the client's immediate needs? Administration of antibiotics Completion of a 12-lead ECG Administration of corticosteroids and bronchodilators Client education: avoidance of irritants like smoke and pollutants

Administration of antibiotics Explanation: Antibiotics are administered to treat respiratory tract infections. Chronic bronchitis is inflammation of the bronchi caused by irritants or infection. Hence, smoking cessation and avoiding pollutants are necessary to slow the accelerated decline of the lung tissue. However, the immediate priority in this case is to cure the infection, pneumonia. Corticosteroids and bronchodilators are administered to asthmatic clients when they show symptoms of wheezing. An ECG is used to evaluate atrial arrhythmias.

Which is a true statement regarding air pressure variances? Air is drawn through the trachea and bronchi into the alveoli during inspiration. Air flows from a region of lower pressure to a region of higher pressure during inspiration. The diaphragm contracts during inspiration. The thoracic cavity becomes smaller during inspiration.

Air is drawn through the trachea and bronchi into the alveoli during inspiration. Explanation: Air flows from a region of higher pressure to a region of lower pressure. During inspiration, movement of the diaphragm and other muscles of respiration enlarge the thoracic cavity, thereby lowering the pressure inside the thorax to a level below that of atmospheric pressure.

What are the primary causes for an acute exacerbation of COPD? Select all that apply. Air pollution Tracheobronchial infection Change in season from spring to summer Gastrointestinal viruses Hypertension

Air pollution Tracheobronchial infection Explanation: Common causes of an acute exacerbation include tracheobronchial infection and air pollution. However, the cause of approximately one third of severe exacerbations cannot be identified. Change in season from spring to summer, hypertension, and GI viruses are not causes of exacerbation of COPD. Winter is worse for COPD when viral and bacterial infections are more prevalent.

A client with severe shortness of breath comes to the emergency department. He tells the emergency department staff that he recently traveled to China for business. Based on his travel history and presentation, the staff suspects severe acute respiratory syndrome (SARS). Which isolation precautions should the staff institute? Droplet precautions Airborne and contact precautions Contact and droplet precautions Contact precautions

Airborne and contact precautions Explanation: SARS, a highly contagious viral respiratory illness, is spread by close person-to-person contact. The client should be placed on airborne and contact precautions to prevent the spread of infection. Droplet precautions don't require a negative air pressure room and wouldn't protect the nurse who touches contaminated items in the client's room. Contact precautions alone don't provide adequate protection from airborne particles.

A patient visited a health care clinic for treatment of upper respiratory tract congestion, fatigue, and sputum production that was rust-colored. Which of the following diagnoses is likely based on this history and inspection of the sputum? Bronchiectasis An infection with pneumococcal pneumonia A lung abscess Bronchitis

An infection with pneumococcal pneumonia Explanation: Sputum that is rust colored suggests infection with pneumococcal pneumonia. Bronchiectasis and a lung abscess usually are associated with purulent thick and yellow-green sputum. Bronchitis usually yields a small amount of purulent sputum.

A patient is brought into the emergency department with carbon monoxide poisoning after escaping a house fire. What should the nurse monitor this patient for? Anemic hypoxia Histotoxic hypoxia Hypoxic hypoxia Stagnant hypoxia

Anemic hypoxia Explanation: Anemic hypoxia is a result of decreased effective hemoglobin concentration, which causes a decrease in the oxygen-carrying capacity of the blood. It is rarely accompanied by hypoxemia. Carbon monoxide poisoning, because it reduces the oxygen-carrying capacity of hemoglobin, produces similar effects but is not strictly anemic hypoxia, because hemoglobin levels may be normal.

The nurse auscultated a patient's middle lobe of the lungs for abnormal breath sounds. To do this, the nurse placed the stethoscope on the: Posterior surface of the left side of the chest, near the sixth rib. Anterior surface of the right side of the chest, between the fourth and fifth rib. Posterior surface of the right side of the chest, near T3. Anterior surface of the left side of the chest, near the sixth rib.

Anterior surface of the right side of the chest, between the fourth and fifth rib. Explanation: The middle lobe of the lung is only found on the right side of the thorax and can only be assessed anteriorly. It is located at the fourth rib, at the right sternal border and extends to the fifth rib, in the midaxillary line. Reference:

What is the most commonly prescribed treatment for the common cold? Antihistamines Decongestants Antitussives Expectorants

Antihistamines Explanation: Antihistamines are the first group of medications recommended for treating sneezing, pruritus, rhinorrhea, and nasal congestion associated with the common cold.

The nurse is mentoring a new graduate nurse. Today, the two of you are caring for a client with a new tracheostomy. The new graduate nurse asks what the complications of tracheostomy are. What would the nurse respond? Select all that apply. Absence of secretions Aspiration Infection Injury to the laryngeal nerve Penetration of the anterior tracheal wall

Aspiration Infection Injury to the laryngeal nerve Explanation: The long-term and short-term complications of tracheostomy include infection, bleeding, airway obstruction resulting from hardened secretions, aspiration, injury to the laryngeal nerve, erosion of the trachea, fistula formation between the esophagus and trachea, and penetration of the posterior tracheal wall.

A client in the intensive care unit has a tracheostomy with humidified oxygen being instilled through it. The client is expectorating thick yellow mucus through the tracheostomy tube frequently. The nurse Sets a schedule to suction the tracheostomy every hour Assesses the client's tracheostomy and lung sounds every 15 minutes Decreases the amount of humidity set to flow through the tracheostomy tube Encourages the client to cough every 30 minutes and prn

Assesses the client's tracheostomy and lung sounds every 15 minutes Explanation: Tracheal suctioning is performed when secretions are obvious or adventitious breath sounds are heard. The client is producing thick yellow mucus frequently, so the nurse needs to make frequent assessments about the need for suctioning. Suctioning every hour could be too frequent or not frequent enough. It also does not address the client's needs. The client needs high humidity to liquify the mucus, which is described as thick. The client has a decresed effectiveness of coughing with a tracheostomy tube. Again, this is not a viable option.

Which ventilator mode provides full ventilatory support by delivering a preset tidal volume and respiratory rate? IMV SIMV Assist control Pressure support

Assist control Explanation: Assist-control ventilation provides full ventilator support by delivering a preset tidal volume and respiratory rate. Intermittent mandatory ventilation (IMV) provides a combination of mechanically assisted breaths and spontaneous breaths. SIMV delivers a preset tidal volume and number of breaths per minute. Between ventilator-delivered breaths, the client can breathe spontaneously with no assistance from the ventilator for those extra breaths. Pressure support ventilation assists SIMV by applying a pressure plateau to the airway throughout the client-triggered inspiration to decrease resistance within the tracheal tube and ventilator tubing.

The nurse is caring for a client reporting chest discomfort. The client's diagnosis at admission is left lower lobe pneumonia. Which strategy will the nurse instruct the client to use to help alleviate the discomfort? Lay on the right side Assume a left side-lying position while in bed Request narcotic medication when pain is experienced Complete deep breathing exercises when chest discomfort occurs

Assume a left side-lying position while in bed Explanation: Pleuritic pain from irritation of the parietal pleura is sharp and seems to "catch" upon inspiration; clients often describe it as "like the stabbing of a knife." Clients are more comfortable when they lay on the affected side because this splints the chest wall, limits expansion and contraction of the lung, and reduces the friction between the injured or diseased pleurae on that side. Pain associated with cough may be reduced manually by splinting the rib cage. The nurse would instruct the client to lay on the left side, not the right, to decrease the pain. While pain medication may be administered, nonpharmacological therapies and nonnarcotic interventions should be implemented first. Deep breathing exercises would not help to decrease the pain, but would rather slow the client's breathing and expand the lungs.

The nurse should be alert for a complication of bronchiectasis that results from a combination of retained secretions and obstruction that leads to the collapse of alveoli. This complication is known as Atelectasis Emphysema Pleurisy Pneumonia

Atelectasis Explanation: Retention of secretions and subsequent obstruction ultimately cause the aveoli distal to the obstruction to collapse (atelectasis).

While assessing an acutely ill client's respiratory rate, the nurse assesses four normal breaths followed by an episode of apnea lasting 20 seconds. How should the nurse document this finding? Eupnea Apnea Biot's respiration Cheyne-Stokes

Biot's respiration Explanation: The nurse will document that the client is demonstrating a Biot's respiration pattern. Biot's respiration is characterized by periods of normal breathing (three to four breaths) followed by varying periods of apnea (usually 10 seconds to 1 minute). Cheyne-Stokes is a similar respiratory pattern, but it involves a regular cycle where the rate and depth of breathing increase and then decrease until apnea occurs. Biot's respiration is not characterized by the increase and decrease in the rate and depth, as characterized by Cheyne-Stokes. Eupnea is a normal breathing pattern of 12 to 18 breaths per minute. Bradypnea is a slower-than-normal rate (<10 breaths per minute), with normal depth and regular rhythm, and no apnea.

What is the difference between cheyne-stokes and biot's breathing?

Biot's respiration is characterized by periods of normal breathing (three to four breaths) followed by varying periods of apnea (usually 10 seconds to 1 minute). Cheyne-Stokes is a similar respiratory pattern, but it involves a regular cycle where the rate and depth of breathing increase and then decrease until apnea occurs. Biot's respiration is not characterized by the increase and decrease in the rate and depth, as characterized by Cheyne-Stokes. Eupnea is a normal breathing pattern of 12 to 18 breaths per minute. Bradypnea is a slower-than-normal rate (<10 breaths per minute), with normal depth and regular rhythm, and no apnea.

Which of the following methods most resembles normal speech following a total laryngectomy? Blom-Singer voice prosthesis Electrolarynx held at neck Esophageal speech Lip speaking

Blom-Singer voice prosthesis Explanation: The Blom-Singer voice prosthesis most resembles normal speech. With esophageal speech, patients compress air into the esophagus and expel it, setting off a vibration of the pharyngeal esophageal segment. With electrolarynx, a battery-powered apparatus projects sound into the oral cavity. When the mouth forms words (articulation), the sounds from the electric larynx becomes audible words. Lip speaking is available during the immediate postoperative period. It does not resemble normal speech.

The nurse is caring for a client being weaned from a mechanical ventilator. Which findings would require the weaning process to be terminated? Blood pressure increase of 20 mm Hg from baseline PaO2 >60 mm Hg with an FiO2 Heart rate Vital capacity of 12 mL/kg

Blood pressure increase of 20 mm Hg from baseline Explanation: In collaboration with the primary provider, the nurse would terminate the weaning process if adverse reactions occur, including a heart rate increase of 20 beats/min, systolic blood pressure increase of 20 mm Hg, a decrease in oxygen saturation to <90%, respiratory rate <8 or >20 breaths/min, ventricular dysrhythmias, fatigue, panic, cyanosis, erratic or labored breathing, and paradoxical chest movement. A vital capacity of 10 to 15 mL/kg, maximum inspiratory pressure (MIP) at least -20 cm H2O, tidal volume of 7 to -9 mL/kg, minute ventilation of 6 L/min, and a rapid/shallow breathing index below 100 breaths/min/L; PaO2 >60 mm Hg with FiO2 <40% are criteria that indicate a client is ready to be weaned from the ventilator. A normal vital capacity is 10 to 15 mL/kg.

Which statement is true about both lung transplant and bullectomy? Both procedures cure COPD. Both procedures treat end-stage emphysema. Both procedures treat patients with bullous emphysema. Both procedures improve the overall quality of life of a client with COPD.

Both procedures improve the overall quality of life of a client with COPD.

In a patient diagnosed with increased intracranial pressure (IICP), the nurse would expect to observe which of the following respiratory rate or depth? Bradypnea Tachypnea Hypoventilation Hyperventilation

Bradypnea Explanation: Bradypnea is a slower than normal rate (<10 breaths/minute), with normal depth and regular rhythm. It is associated with IICP, brain injury, central nervous system depressants, and drug overdose. Tachypnea is associated with metabolic acidosis, septicemia, severe pain, and rib fracture. Hypoventilation is shallow, irregular breathing. Hyperventilation is an increased rate and depth of breathing.

The nurse is caring for a client following a thoracotomy. Which finding requires immediate intervention by the nurse? Heart rate, 112 bpm Moderate amounts of colorless sputum Pain of 5 on a 1-to-10 scale Chest tube drainage, 190 mL/hr

Chest tube drainage, 190 mL/hr Explanation: The nurse should monitor and document the amount and character of drainage every 2 hours. The nurse must notify the primary provider if drainage is ≥150 mL/hr. The other findings are normal following a thoracotomy and no intervention would be required.

The nurse is caring for a client in the ICU who is receiving mechanical ventilation. Which nursing measure is implemented in an effort to reduce the client's risk of developing ventilator-associated pneumonia (VAP)? Cleaning the client's mouth with chlorhexidine daily Maintaining the client in a high Fowler's position Ensuring that the client remains sedated while intubated Turning and repositioning the client every 4 hours

Cleaning the client's mouth with chlorhexidine daily Explanation: The five key elements of the VAP bundle include elevation of the head of the bed (30 to 45 degrees [semi-Fowler's position)], daily "sedation vacations," and assessment of readiness to extubate; peptic ulcer disease prophylaxis (with histamine-2 receptor antagonists, such as ranitidine [Zantac]); deep venous thrombosis prophylaxis; and daily oral care with chlorhexidine (0.12% oral rinses). The client should be turned and repositioned every 2 hours to prevent complications of immobility and atelectasis and to optimize lung expansion.

A client with lung cancer develops pleural effusion. During chest auscultation, which breath sound should the nurse expect to hear? Crackles Rhonchi Decreased breath sounds Wheezes

Decreased breath sounds Explanation: In pleural effusion, fluid accumulates in the pleural space, impairing transmission of normal breath sounds. Because of the acoustic mismatch, breath sounds are diminished. Crackles commonly accompany atelectasis, interstitial fibrosis, and left-sided heart failure. Rhonchi suggest secretions in the large airways. Wheezes result from narrowed airways, such as in asthma, chronic obstructive pulmonary disease, and bronchitis. Reference:

A home health nurse sees a client with end-stage chronic obstructive pulmonary disease. An outcome identified for this client is preventing infection. Which finding indicates that this outcome has been met? Decreased oxygen requirements Increased sputum production Decreased activity tolerance Normothermia

Decreased oxygen requirements Explanation: A client who is free from infection will most likely have decreased oxygen requirements. A client with infection will display increased sputum production, fever, shortness of breath, decreased activity tolerance, and increased oxygen requirements.

Which is an age-related change associated with the respiratory system? Increased chest muscle mass Thinning of alveolar membranes Decreased size of the airway Increased elasticity of alveolar sacs

Decreased size of the airway Explanation: Age-related changes that occur in the respiratory system are a decrease in the size of the airway, decreased chest muscle mass, increased thickening of the alveolar membranes, and decreased elasticity of the alveolar sacs.

A client who has recently started working in a coal mine is concerned the effects on long-term health. How does the nurse advise the client to prevent occupational lung disease? Select all that apply. Do not smoke, or quit smoking if currently smoking. Wear appropriate protective equipment when around airborne irritants and dusts. Try to find another occupation as soon as possible. Schedule an annual lung x-ray to monitor health.

Do not smoke, or quit smoking if currently smoking. Wear appropriate protective equipment when around airborne irritants and dusts. Explanation: The nurse may instruct clients that the following precautions may help prevent occupational lung disease: not smoking, wearing appropriate protective equipment when around airborne irritants and dusts, scheduling lung function evaluation with spirometry as recommended, becoming educated about lung diseases, and paying attention to risk evaluation of the workplace to identify risks for lung disease.

The diagnosis of pulmonary hypertension associated with chronic obstructive pulmonary disease (COPD) is suspected when which of the following is noted? Select all that apply. Dyspnea and fatigue disproportionate to pulmonary function abnormalities Right ventricular enlargement Elevated plasma brain natriuretic peptide (BNP) Enlargement of central pulmonary arteries Left ventricular hypertrophy

Dyspnea and fatigue disproportionate to pulmonary function abnormalities Right ventricular enlargement Elevated plasma brain natriuretic peptide (BNP) Enlargement of central pulmonary arteries Explanation: The diagnosis of pulmonary hypertension associated with COPD is suspected in patients complaining of dyspnea and fatigue that appear to be disproportionate to pulmonary function abnormalities. Enlargement of the central pulmonary arteries on the chest X-ray, echocardiogram suggestive of right ventricular enlargement, and elevated plasma BNP may be present.

A nurse is assessing a client who comes to the clinic for care. Which findings in this client suggest bacterial pneumonia? Nonproductive cough and normal temperature Sore throat and abdominal pain Hemoptysis and dysuria Dyspnea and wheezing

Dyspnea and wheezing Explanation: In a client with bacterial pneumonia, retained secretions cause dyspnea, and respiratory tract inflammation causes wheezing. Bacterial pneumonia also produces a productive cough and fever, rather than a nonproductive cough and normal temperature. Sore throat occurs in pharyngitis, not bacterial pneumonia. Abdominal pain is characteristic of a GI disorder, unlike chest pain, which can reflect a respiratory infection such as pneumonia. Hemoptysis and dysuria aren't associated with pneumonia.

A client has suspected fluid accumulation in the pleural space of the lungs and is scheduled for a thoracentesis. The nurse will implement which of the following for this procedure? Select all that apply. Place the client in the prone position. Educate the client about the need to cleanse the thoracic area. Apply pressure to the puncture site after the procedure. Prepare the client for magnetic resonance imaging after the procedure to verify tube placement. Complete a respiratory assessment after the procedure.

Educate the client about the need to cleanse the thoracic area. Apply pressure to the puncture site after the procedure. Complete a respiratory assessment after the procedure. Explanation: A thoracentesis is performed to aspirate fluid or air from the pleural space. The nurse assists the client to a sitting or side-lying position, which provides support and exposes the base of the thorax. Encouraging a position of comfort helps the client to relax for the procedure. The nurse prepares the client by explaining the steps of the procedure and begins by cleansing the thoracic area using aseptic technique. After the procedure, the nurse applies pressure to the site to help stop bleeding; then, he or she applies an air-tight, sterile dressing. A chest x-ray verifies that there is no pneumonthorax. The nurse will monitor at intervals the client's respiratory function.

The nurse is assessing a patient who has been admitted with possible ARDS. What findings would distinguish ARDS from cardiogenic pulmonary edema? Elevated white blood count Elevated troponin levels Elevated myoglobin levels Elevated B-type natriuretic peptide (BNP) levels

Elevated B-type natriuretic peptide (BNP) levels Explanation: Common diagnostic tests performed in patients with potential ARDS include plasma brain natriuretic peptide (BNP) levels, echocardiography, and pulmonary artery catheterization. The BNP level is helpful in distinguishing ARDS from cardiogenic pulmonary edema.

High or increased compliance occurs in which condition? Emphysema Pneumothorax Pleural effusion ARDS

Emphysema Explanation: High or increased compliance occurs if the lungs have lost their elasticity and the thorax is overdistended, as in emphysema. Conditions associated with decreased compliance include pneumothorax, pleural effusion, and acute respiratory distress syndrome (ARDS).

A patient prescribed a medication for hypertension started taking it 3 days ago and arrives in the emergency department with an edematous face and tongue and having a difficult time speaking. What medication is the nurse aware of that may produce this type of side effect? Metoprolol succinate (Toprol XL) Amlodipine (Norvasc) Enalapril (Vasotec) Valsartan (Diovan)

Enalapril (Vasotec) Explanation: Use of ACE inhibitors, such as enalapril (Vasotec), is a risk factor for laryngeal obstruction resulting from edema of the throat as a result of a side effect of the drug.

The nurse is planning the care for a client at risk of developing pulmonary embolism. What nursing interventions should be included in the care plan? Select all that apply. Encouraging a liberal fluid intake Instructing the client to move the legs in a "pumping" exercise Instructing the client to dangle the legs over the side of the bed for 30 minutes, four times a day Using elastic stockings, especially when decreased mobility would promote venous stasis Applying a sequential compression device

Encouraging a liberal fluid intake Instructing the client to move the legs in a "pumping" exercise Using elastic stockings, especially when decreased mobility would promote venous stasis Applying a sequential compression device Explanation: The use of anti-embolism stockings or intermittent pneumatic leg compression devices reduces venous stasis. These measures compress the superficial veins and increase the velocity of blood in the deep veins by redirecting the blood through the deep veins. Having the client move the legs in a "pumping" exercise helps increase venous flow. Legs should not be dangled or feet placed in a dependent position while the client sits on the edge of the bed; instead, feet should rest on the floor or on a chair.

The nurse is educating a patient diagnosed with acute bacterial rhinosinusitis about interventions that may assist with symptom control. What should the nurse include in this information? (Select all that apply.) Take an over-the-counter nasal decongestant. Take an over-the-counter antihistamine. Ensure an adequate fluid intake. Increase the humidity in the home. Apply local heat to promote drainage.

Ensure an adequate fluid intake. Increase the humidity in the home. Apply local heat to promote drainage.

A nurse notes that the FEV1/FVC ratio is less than 70% and the FEV1 is 65% for a patient with COPD. What stage should the nurse document the patient is in? I II III IV

II Explanation: All grades of COPD are associated with an FEV1/FVC ratio of less than 70%. Grade I (mild) is associated with an FEV1 of greater than or equal to 80%. Grade II (moderate) is associated with an FEV1 of 50%-80%. Grade III is associated with an FEV1 of <30%-50%. Grade IV is associated with an FEV1 of <30%.

A client diagnosed with asthma is preparing for discharge. The nurse is educating the client on the proper use of a peak flow meter. The nurse instructs the client to complete which action? Move the indicator to the top of the numbered scale. Sit down while completing a peak flow reading. Take and record peak flow readings three times daily. If coughing occurs during the procedure, repeat it.

If coughing occurs during the procedure, repeat it. Explanation: Steps for using the peak flow meter correctly include (1) moving the indicator to the bottom of the numbered scale; (2) standing up; (3) taking a deep breath and filling the lungs completely; (4) placing the mouthpiece in the mouth and closing the lips around it; (5) blowing out hard and fast with a single blow; and (6) recording the number achieved on the indicator. If the client coughs or a mistake is made in the process, repeat the procedure. Peak flow readings should be taken during an asthma attack.

The nurse has instructed a client on how to perform pursed-lip breathing. The nurse recognizes the purpose of this type of breathing is to accomplish which result? Promote more efficient and controlled ventilation and to decrease the work of breathing Improve oxygen transport; induce a slow, deep breathing pattern; and assist the client to control breathing Promote the strengthening of the client's diaphragm Promote the client's ability to take in oxygen

Improve oxygen transport; induce a slow, deep breathing pattern; and assist the client to control breathing

A nurse is assisting a client with mild chronic obstructive pulmonary disease (COPD) to set a goal related to the condition. Which of the following is an appropriate goal for this client? Maintain activity level of walking to the mailbox. Continue with current level of mobility at home. Increase walking distance around a city block without shortness of breath. Relieve shortness of breath to a level as close as possible to tolerable.

Increase walking distance around a city block without shortness of breath. Explanation: If the client has mild COPD, goals are to increase exercise and prevent further loss of pulmonary function. The client who increases his walking distance without shortness of breath meets these criteria. If the client has severe COPD, goals are then to preserve current pulmonary function and relieve symptoms as much as possible. Examples of these goals are the other options, in which the activity level is at current and symptoms are relieved to tolerable or close to tolerable. `

Which measure may increase complications for a client with COPD? Administration of antibiotics Increased oxygen supply Administration of antitussive agents Decreased oxygen supply

Increased oxygen supply Explanation: Administering too much oxygen can result in the retention of carbon dioxide. Clients with alveolar hypoventilation cannot increase ventilation to adjust for this increased load, and hypercapnia occurs. All the other measures aim to prevent complications.

The nurse is planning for the care of a client with acute tracheobronchitis. What nursing interventions should be included in the plan of care? Select all that apply. Increasing fluid intake to remove secretions Encouraging the client to rest Using cool-vapor therapy to relieve laryngeal and tracheal irritation Giving 3 L fluid per day Administering a narcotic analgesic for pain

Increasing fluid intake to remove secretions Encouraging the client to rest Using cool-vapor therapy to relieve laryngeal and tracheal irritation Explanation: In most cases, treatment of tracheobronchitis is largely symptomatic. Cool vapor therapy or steam inhalations may help relieve laryngeal and tracheal irritation. A primary nursing function is to encourage bronchial hygiene, such as increased fluid intake and directed coughing to remove secretions. Fatigue is a consequence of tracheobronchitis; therefore, the nurse cautions the client against overexertion, which can induce a relapse or exacerbation of the infection. The client is advised to rest.

A nursing student understands the importance of the psychosocial aspects of disease processes. When working with a patient with COPD, the student would rank which of the following nursing diagnoses as the MOST important when analyzing the psychosocial effects? Disturbed sleep pattern related to cough Ineffective coping related to anxiety High risk for ineffective therapeutic regimen management related to lack of knowledge Activity intolerance related to fatigue

Ineffective coping related to anxiety Explanation: Any factor that interferes with normal breathing quite naturally induces anxiety, depression, and changes in behavior. Constant shortness of breath and fatigue may make the patient irritable and apprehensive to the point of panic. Although the other choices are correct, the most important psychosocial nursing diagnosis for a patient with COPD is ineffective coping related to a high level of anxiety. Reference:

The nurse is instructing the patient on the collection of a sputum specimen. What should be included in the instructions? (Select all that apply.) Initially, clear the nose and throat. Spit surface mucus and saliva into a sterile specimen container. Take a few deep breaths before coughing. Use diaphragmatic contractions to aid in the expulsion of sputum. Rinse with mouthwash prior to providing the specimen.

Initially, clear the nose and throat. Take a few deep breaths before coughing. Use diaphragmatic contractions to aid in the expulsion of sputum. Explanation: Barrel chest occurs as a result of overinflation of the lungs, which increases the anteroposterior diameter of the thorax. It occurs with aging and is a hallmark sign of emphysema and COPD. In a patient with emphysema, the ribs are more widely spaced and the intercostal spaces tend to bulge on expiration. Funnel chest occurs when there is a depression in the lower portion of the sternum. This may compress the heart and great vessels, resulting in murmurs. Funnel chest may occur with rickets or Marfan's syndrome. A pigeon chest occurs as a result of the anterior displacement of the sternum, which also increases the anteroposterior diameter. This may occur with rickets, Marfan syndrome, or severe kyphoscoliosis. Kyphoscoliosis is characterized by elevation of the scapula and a corresponding S-shaped spine. This deformity limits lung expansion within the thorax. It may occur with osteoporosis and other skeletal disorders that affect the thorax.

The nurse is caring for a client who underwent a laryngectomy. Which intervention will the nurse initially complete in an effort to meet the client's nutritional needs? Initiate enteral feedings. Offer plenty of thin liquids. Encourage sweet foods. Liberally season foods.

Initiate enteral feedings. Explanation: Postoperatively, the client may not be permitted to eat or drink for at least 7 days. Alternative sources of nutrition and hydration include IV fluids, enteral feedings through a nasogastric or gastrostomy tube, and parenteral nutrition. Once the client is permitted to resume oral feedings, thick liquids are offered; sweet foods are avoided because they cause increased salivation and decrease the client's appetite. The client's taste sensations are altered for a while after surgery because inhaled air passes directly into the trachea, bypassing the nose and the olfactory end organs. In time, however, the client usually accommodates to this change and olfactory sensation adapts; thus, seasoning is based on personal preferences.

A pediatrician diagnosed a child with swollen and inflamed adenoids. The nurse practitioner confirmed the diagnosis by: Palpating the throat above the cricoid cartilage. Inspecting the roof of the nasopharynx. Examining the base of the oropharynx. Inspecting the posterior region of the epiglottis.

Inspecting the roof of the nasopharynx. Explanation: The adenoids are clusters of lymph tissue located between the back of the nose and the back of the throat in the nasopharynx. The adenoids are usually inspected by using a special mirror. They cannot be seen by looking directly into the mouth.

A new nurse auscultates adventitious breath sounds but is not sure what to document and confers with an experienced nurse. This experienced nurse documents a pleural friction rub. Which of the following did the experienced nurse do during her assessment to identify the rub? Instructed the client to cough Instructed the client to hold the breath Listened over the upper posterior lung surface Used percussion to verify the sounds

Instructed the client to hold the breath Explanation: Inflammation of the pleural lining can cause a grating, harsh, crackling sound that disappears when the client holds the breath. Coughing does not clear the rub. Rubs are best heard over the lower lateral anterior surface of the thorax.

Which ventilator mode provides a combination of mechanically assisted breaths and spontaneous breaths? Intermittent mandatory ventilation (IMV) Assist control Synchronized intermittent mandatory ventilation (SIMV) Pressure support

Intermittent mandatory ventilation (IMV) Explanation: IMV provides a combination of mechanically assisted breaths and spontaneous breaths. Assist-control ventilation provides full ventilator support by delivering a preset tidal volume and respiratory rate. SIMV delivers a preset tidal volume and number of breaths per minute. Between ventilator-delivered breaths, the client can breathe spontaneously with no assistance from the ventilator for those extra breaths. Pressure support ventilation assists SIMV by applying a pressure plateau to the airway throughout the client-triggered inspiration to decrease resistance within the tracheal tube and ventilator tubing.

Which ventilator mode provides a combination of mechanically assisted breaths and spontaneous breaths? Intermittent mandatory ventilation (IMV) Assist control Synchronized intermittent mandatory ventilation (SIMV) Pressure support

Intermittent mandatory ventilation (IMV) Explanation: IMV provides a combination of mechanically assisted breaths and spontaneous breaths. Assist-control ventilation provides full ventilator support by delivering a preset tidal volume and respiratory rate. SIMV delivers a preset tidal volume and number of breaths per minute. Between ventilator-delivered breaths, the client can breathe spontaneously with no assistance from the ventilator for those extra breaths. Pressure support ventilation assists SIMV by applying a pressure plateau to the airway throughout the client-triggered inspiration to decrease resistance within the tracheal tube and ventilator tubing. Reference:

What happens to the diaphragm during inspiration? It relaxes and raises. It contracts and flattens. It relaxes and flattens. It contracts and raises.

It contracts and flattens. Explanation: During inspiration, the diaphragm contracts and flattens, which expands the thoracic cage and increases the thoracic cavity. Reference:

A nurse is teaching a client with emphysema how to perform pursed-lip breathing. The client asks the nurse to explain the purpose of this breathing technique. Which explanation should the nurse provide? It helps prevent early airway collapse. It increases inspiratory muscle strength. It decreases use of accessory breathing muscles. It prolongs the inspiratory phase of respiration.

It helps prevent early airway collapse. Explanation: Pursed-lip breathing helps prevent early airway collapse. Learning this technique helps the client control respiration during periods of excitement, anxiety, exercise, and respiratory distress. To increase inspiratory muscle strength and endurance, the client may need to learn inspiratory resistive breathing. To decrease accessory muscle use and thus reduce the work of breathing, the client may need to learn diaphragmatic (abdominal) breathing. In pursed-lip breathing, the client mimics a normal inspiratory-expiratory (I:E) ratio of 1:2. (A client with emphysema may have an I:E ratio as high as 1:4.)

The nurse is educating a patient with COPD about the technique for performing pursed-lip breathing. What does the nurse inform the patient is the importance of using this technique? It prolongs exhalation. It increases the respiratory rate to improve oxygenation. It will assist with widening the airway. It will prevent the alveoli from overexpanding.

It prolongs exhalation. Explanation: The goal of pursed-lip breathing is to prolong exhalation and increase airway pressure during expiration, thus reducing the amount of trapped air and the amount of airway resistance.

The nurse is instructing a client who is scheduled for a laryngectomy about methods of laryngeal speech. Which best describes tracheoesophageal puncture (TEP)? It requires the insertion of a prosthesis into the trachea. It requires the client to hold a throat vibrator against the neck. It enables the client to form words with the lips. It will result in a low, gruff-sounding voice.

It requires the insertion of a prosthesis into the trachea. Explanation: TEP requires a surgical opening in the posterior wall of the trachea, followed by the insertion of a prosthesis such as a Blom-Singer device. An artificial larynx is a throat vibrator held against the neck that projects sound into the mouth. With esophageal speech, the client forms words with the lips. Esophageal speech causes the voice quality to be lower pitched and gruff sounding.

The nursing instructor is teaching students about types of lung cancer. Which type of lung cancer is characterized as fast growing and can arise peripherally? Large cell carcinoma Bronchoalveolar carcinoma Adenocarcinoma Squamous cell carcinoma

Large cell carcinoma Explanation: Large cell carcinoma is a fast-growing tumor that tends to arise peripherally. Bronchoalveolar cell cancer arises from the terminal bronchus and alveoli and usually grows slowly. Adenocarcinoma presents as peripheral masses or nodules and often metastasizes. Squamous cell carcinoma arises from the bronchial epithelium and is more centrally located.

Late complications of radiation therapy may include which of the following? Select all that apply. Laryngeal necrosis Edema Fibrosis Loss of taste Xerostomia

Laryngeal necrosis Edema Fibrosis Explanation: Complications occurring late may include laryngeal necrosis, edema, and fibrosis. Loss of taste and xerostomia are symptoms of radiation therapy that may occur earlier in treatment.

A client presents to the ED reporting severe coughing episodes. The client states that "the episodes are more intense at night." The nurse should suspect which of the following conditions based on the client's primary report? Left-sided heart failure Chronic obstructive pulmonary disorder (COPD) Emphysema Bronchitis

Left-sided heart failure Explanation: Coughing at night may indicate the onset of left-sided heart failure or bronchial asthma. A cough in the morning with sputum production may indicate bronchitis. A cough that worsens when the client is supine suggests postnasal drip (rhinosinusitis). Coughing after food intake may indicate aspiration of material into the tracheobronchial tree. A cough with recent onset is usually caused by an acute infection. A cough that occurs more frequently at night is not associated with COPD, emphysema, or bronchitis.

Which community-acquired pneumonia demonstrates the highest occurrence during summer and fall? Legionnaires disease Streptococcal (pneumococcal) pneumonia Mycoplasmata pneumonia Viral pneumonia

Legionnaires disease accounts for 15% of community-acquired pneumonias; it occurs mainly in summer and fall. Streptococcal and viral pneumonias demonstrate the highest occurrence during the winter months. Mycoplasmal pneumonia demonstrates the highest occurrence in fall and early winter.

The nurse is working on a busy respiratory unit. In caring for a variety of clients, the nurse must be knowledgeable of diagnostic studies. With which diagnostic studies would the nurse screen the client for an allergy to iodine? Select all that apply. Lung scan Chest x-ray Fluoroscopy Pulmonary angiography Bronchoscopy Pulmonary functions test

Lung scan Fluoroscopy Pulmonary angiography Explanation: The nurse must be well educated in screening clients before diagnostic procedures which include contrast medium for an allergy to iodine. A lung scan, fluoroscopy and pulmonary angiography all require contrast medium.

Which oxygen administration device has the advantage of providing a high oxygen concentration? Nonrebreathing mask Venturi mask Catheter Face tent

Nonrebreathing mask Explanation: Nonrebreathing masks provide high oxygen concentrations but usually fit poorly. A Venturi mask provides low levels of supplemental oxygen. A catheter is an inexpensive device that provides a variable fraction of inspired oxygen and may cause gastric distention. A face tent provides a fairly accurate fraction of inspired oxygen but is bulky and uncomfortable. It would not be the device of choice to provide a high oxygen concentration.

Which type of sleep apnea is characterized by lack of airflow due to pharyngeal occlusion? Simple Mixed Obstructive Central

Obstructive Explanation: Obstructive sleep apnea occurs usually in men, especially men who are older and overweight. Types of sleep apnea do not include a simple classification. Mixed sleep apnea is a combination of central and obstructive apnea, with one apneic episode. In central sleep apnea, the client demonstrates simultaneous cessation of both airflow and respiratory movements.

A client is postoperative for a partial laryngectomy following a diagnosed malignancy. The client is to start oral feedings. The nurse does the following interventions: (Select all that apply.) Facilitates privacy while eating Obtains results of a swallow study Provides thick liquids Orders a regular diet tray Encourages the client to ingest sweet foods

Obtains results of a swallow study Provides thick liquids Explanation: When a client is allowed to eat following a partial laryngectomy, a swallow study may be obtained first to determine the client's risk of aspiration. The client is started with thick liquids because they are easy to swallow. The nurse stays with the client during initial feedings to ensure safe ingestion. Solid foods are introduced as tolerated. The nurse encourages the client to avoid sweet foods, which increase salivation and suppress appetite.

The nurse is caring for a client with hypoxemia of unknown cause. Which of the following oxygen transport considerations does the nurse identify as crucial to circulate oxygen in the body system? Select all that apply. Oxygen is dissolved. High blood pressure disrupts oxygen transport. Oxyhemoglobin circulates to the body tissue. All systemic oxygen is available for diffusion. Adequate red blood cells are needed for oxygen transport.

Oxygen is dissolved. Oxyhemoglobin circulates to the body tissue. Adequate red blood cells are needed for oxygen transport. Explanation: Oxygen transport occurs by dissolving oxygen in the water in the plasma and combining oxygen with red blood cells (oxyhemoglobin). Normal red blood cell count is needed for oxygen transport. High blood pressure does not disrupt transport unless there is disruption in perfusion via a bleeding or occlusion. Dissolved oxygen is the only form which can diffuse across cell membranes.

A young adult visited a clinic because he was injured during a softball game. He told the nurse that the ball struck him in his "Adam's apple." To assess the initial impact of injury, the nurse: Inspects the vocal cords. Inspects the epiglottis. Palpates the thyroid cartilage. Palpates the cricoid cartilage.

Palpates the thyroid cartilage. Explanation: The term "Adam's Apple" is used to refer to a lump or protrusion, a laryngeal prominence. It is formed by the angle of the thyroid cartilage surrounding the larynx.

A patient is diagnosed as being in the early stage of laryngeal cancer of the glottis with only 1 vocal cord involved. For what type of surgical intervention will the nurse plan to provide education? Total laryngectomy Cordectomy Vocal cord stripping Partial laryngectomy

Partial laryngectomy Explanation: A partial laryngectomy (laryngofissure-thyrotomy) is often used for patients in the early stages of cancer in the glottis area when only one vocal cord is involved. Reference:

A client has hypoxemia of pulmonary origin. What portion of arterial blood gas results is most useful in distinguishing between acute respiratory distress syndrome and acute respiratory failure? Partial pressure of arterial oxygen (PaO2) Partial pressure of arterial carbon dioxide (PaCO2) pH Bicarbonate (HCO3-)

Partial pressure of arterial oxygen (PaO2) Explanation: In acute respiratory failure, administering supplemental oxygen elevates the PaO2. In acute respiratory distress syndrome, elevation of the PaO2 requires positive end-expiratory pressure. In both situations, the PaCO2 is elevated and the pH and HCO3- are depressed.

The nurse is assessing a client in the clinic, and upon physical assessment the client demonstrates displacement of the sternum. This would be documented as which condition? Barrel chest Funnel chest Kyphoscoliosis Pigeon chest

Pigeon chest Explanation: Pigeon chest may occur with rickets, Marfan syndrome, or severe kyphoscoliosis. A barrel chest is seen in clients with emphysema and occurs as a result of overinflation of the lungs. A funnel chest occurs when there is a depression in the lower portion of the sternum. Kyphoscoliosis is characterized by elevation of the scapula and a corresponding S-shaped spine.

During a preadmission assessment, the nurse finds increased tactile fremitus. She knows this sign is consistent with which of the following diagnoses? Bronchitis Emphysema Atelectasis Pneumonia

Pneumonia Explanation: Air does not conduct sound well, but a solid substance such as a tumor, or fluid that increases the density of the lung, as occurs in pneumonia, does. Therefore, an increase in solid tissue of the normally air-filled lung enhances fremitus, and an increase in air in the lung impedes sound. A patient with consolidation of a lobe of the lung from pneumonia has increased tactile fremitus over that lobe.

The client is prescribed albuterol 2 puffs as a metered-dose inhaler. Which action by the client demonstrates understanding of administration for this medication? Positions the inhaler 2 finger widths away from the lips Carefully holds the inhaler upright without shaking it Holds the breath for 5 seconds after administering the medication Immediately repeats the second puff after the first puff

Positions the inhaler 2 finger widths away from the lips Explanation: To administer a metered-dose inhaler, the client holds the inhaler upright and shakes the inhaler. The inhaler is positioned 2 finger widths away from lips. After administering the medication, the client holds the breath for as long as possible, at least 10 seconds. The client may administer the next puff in 15 to 30 seconds.

The nurse is educating a patient with asthma about preventative measures to avoid having an asthma attack. What does the nurse inform the patient is a priority intervention to prevent an asthma attack? Using a long-acting steroid inhaler when an attack is coming Avoiding exercise and any strenuous activity Preparing a written action plan Staying in the house if it is too cold or too hot

Preparing a written action plan Explanation: Asthma exacerbations are best managed by early treatment and education, including the use of written action plans as part of any overall effort to educate patients about self-management techniques, especially those with moderate or severe persistent asthma or with a history of severe exacerbations (Expert Panel Report 3, 2007).

The nurse is performing a nutritional assessment on a client who has been diagnosed with cancer of the larynx. Which laboratory values would be assessed when determining the nutritional status of the client? Select all that apply. White blood cell count Protein level Albumin level Platelet count Glucose level

Protein level Albumin level Glucose level Explanation: The nurse also assesses the client's general state of nutrition, including height and weight and body mass index, and reviews laboratory values that assist in determining the client's nutritional status (albumin, protein, glucose, and electrolyte levels). The white blood cell count and the platelet count would not normally assist in determining the client's nutritional status.

A client presents to the ED experiencing symptoms of COPD exacerbation. The nurse understands that goals of therapy should be achieved to improve the client's condition. Which statements reflect therapy goals? Select all that apply. Provide medical support for the current exacerbation. Treat the underlying cause of the event. Return the client to their original functioning abilities. Provide long-term support for medical management. Teach the client to suspend activity.

Provide medical support for the current exacerbation. Treat the underlying cause of the event. Return the client to their original functioning abilities. Provide long-term support for medical management. Explanation: The goal is to have a stable client with COPD leading the most productive life possible. COPD cannot necessarily be cured, but it can be managed so that the client can live a reasonably normal life. With adequate management, clients should not have to give up their usual activities.

Which diagnostic test is most accurate in assessing acute airway obstruction? Arterial blood gases (ABGs) Pulmonary function studies Pulse oximetry Spirometry

Pulmonary function studies Explanation: Pulmonary function studies are used to help confirm the diagnosis of COPD, determine disease severity, and monitor disease progression. ABGs and pulse oximetry are not the most accurate diagnostics for an airway obstruction. Spirometry is used to evaluate airflow obstruction, which is determined by the ratio of FEV1 to forced vital capacity (FVC).

Which action by the nurse is most appropriate when the client demonstrates subcutaneous emphysema along the suture line or chest dressing 2 hours after chest surgery? Apply a compression dressing to the area Measure the patient's pulse oximetry Report the finding to the physician immediately Record the observation

Record the observation Explanation: The nurse should record the observation. Subcutaneous emphysema is a typical finding in clients after chest surgery. Subcutaneous emphysema occurs after chest surgery as the air that is located within the pleural cavity is expelled through the tissue opening created by the surgical procedure. Subcutaneous emphysema is absorbed by the body spontaneously after the underlying leak is treated or halted. Subcutaneous emphysema results from air entering the tissue planes. It is unnecessary to report the finding to the physician or apply a compression dressing because subcutaneous emphysema is an expected finding at this stage of recovery. Subcutaneous emphysema is not an explicit risk factor for hypoxemia, so no extraordinary monitoring of pulse oximetry is necessary.

Positive end-expiratory pressure (PEEP) therapy has which effect on the heart? Bradycardia Tachycardia Increased blood pressure Reduced cardiac output

Reduced cardiac output Explanation: PEEP reduces cardiac output by increasing intrathoracic pressure and reducing the amount of blood delivered to the left side of the heart. It doesn't affect heart rate, but a decrease in cardiac output may reduce blood pressure, commonly causing compensatory tachycardia, not bradycardia. However, the resulting tachycardia isn't a direct effect of PEEP therapy itself. Reference:

A student nurse is working with a client who is diagnosed with head trauma. The nurse has documented Cheyne-Stokes respirations. The student would expect to see which of the following? Irregular breathing at 14 to 18 breaths per minute Period of cessation of breathing Periods of normal breathing followed by periods of apnea Regular breathing where the rate and depth increase, then decrease

Regular breathing where the rate and depth increase, then decrease Explanation: Observing the rate and depth of respiration is an important aspect of the nursing assessment. Certain patterns of breathing are characteristic of specific disease states or conditions. Head trauma can cause damage to the respiratory center in the brain, thereby altering the rate and depth of respirations. Cheyne-Stokes breathing is characterized by a regular cycle in which the rate and depth of breathing increase, then decrease until apnea occurs. Reference:

A patient is being treated for status asthmaticus. What danger sign does the nurse observe that can indicate impending respiratory failure? Respiratory acidosis Respiratory alkalosis Metabolic acidosis Metabolic alkalosis

Respiratory acidosis Explanation: In status asthmaticus, increasing PaCO2 (to normal levels or levels indicating respiratory acidosis) is a danger sign signifying impending respiratory failure. Understanding the sequence of the pathophysiologic processes in status asthmaticus is important for understanding assessment findings. Respiratory alkalosis occurs initially because the patient hyperventilates and PaCO2 decreases. As the condition continues, air becomes trapped in the narrowed airways and carbon dioxide is retained, leading to respiratory acidosis.

The nurse, caring for a patient with emphysema, understands that airflow limitations are not reversible. The end result of deterioration is: Diminished alveolar surface area. Hypercapnia resulting from decreased carbon dioxide elimination. Hypoxemia secondary to impaired oxygen diffusion. Respiratory acidosis.

Respiratory acidosis. Explanation: Decreased carbon dioxide elimination results in increased carbon dioxide

A nurse administers albuterol (Proventil), as ordered, to a client with emphysema. Which finding indicates that the drug is producing a therapeutic effect? Respiratory rate of 22 breaths/minute Dilated and reactive pupils Urine output of 40 ml/hour Heart rate of 100 beats/minute

Respiratory rate of 22 breaths/minute Correct response: Respiratory rate of 22 breaths/minute Explanation: In a client with emphysema, albuterol is used as a bronchodilator. A respiratory rate of 22 breaths/minute indicates that the drug has achieved its therapeutic effect because fewer respirations are required to achieve oxygenation. Albuterol has no effect on pupil reaction or urine output. It may cause a change in the heart rate, but this is an adverse, not therapeutic, effect.

A pneumothorax is a possible complication of COPD. Symptoms will depend on the suddenness of the attack and the size of the air leak. The most common, immediate symptom that should be assessed is: Sharp, stabbing chest pain Dyspnea A dry, hacking cough Tachycardia

Sharp, stabbing chest pain Explanation: The initial symptom is usually chest pain of sudden onset that leads to feelings of chest pressure, dyspnea, and tachycardia. A cough may be present.

he nurse is teaching a postoperative client who had a coronary artery bypass graft about using the incentive spirometer. The nurse instructs the client to perform the exercise in the following order: 1Sit in an upright position. 2Place the mouthpiece of the spirometer in the mouth. 3Breathe air in through the mouth. 4Hold breath for about 3 seconds. 5Exhale air slowly through the mouth.

Sit in an upright position. Place the mouthpiece of the spirometer in the mouth. Breathe air in through the mouth. Hold breath for about 3 seconds. Exhale air slowly through the mouth. Explanation: The nurse instructs the client, when using the incentive spirometer, the proper use of it. First, the client is to sit in an upright position. The client is then to place the mouthpiece of the spirometer in the client's mouth. Next, the client breathes air in through the mouth. This causes the incentive spirometer to be activated. The client holds his breath for about 3 seconds. Then, the client exhales slowly through the mouth. Reference:

An adult client has just been diagnosed with small cell lung cancer. The client asks the nurse why the doctor is not offering surgery as a treatment for his cancer. What fact about lung cancer treatment should inform the nurse's response? The cells in small cell cancer of the lung are not large enough to visualize in surgery. Small cell lung cancer is self-limiting in many clients and surgery should be delayed. Clients with small cell lung cancer are not normally stable enough to survive surgery. Small cell cancer of the lung grows rapidly and metastasizes early and extensively.

Small cell cancer of the lung grows rapidly and metastasizes early and extensively.

The nurse is caring for a patient with pleurisy. What symptoms does the nurse recognize are significant for this patient's diagnosis? Dullness or flatness on percussion over areas of collected fluid Dyspnea and coughing Fever and chills Stabbing pain during respiratory movement

Stabbing pain during respiratory movement Explanation: When the inflamed pleural membranes rub together during respiration (intensified on inspiration), the result is severe, sharp, knifelike pain. The key characteristic of pleuritic pain is its relationship to respiratory movement. Taking a deep breath, coughing, or sneezing worsens the pain. Pleuritic pain is limited in distribution rather than diffuse; it usually occurs only on one side. The pain may become minimal or absent when the breath is held. It may be localized or radiate to the shoulder or abdomen. Later, as pleural fluid develops, the pain decreases.

A patient comes to the clinic with fever, cough, and chest discomfort. The nurse auscultates crackles in the left lower base of the lung and suspects that the patient may have pneumonia. What does the nurse know is the most common organism that causes community-acquired pneumonia? Staphylococcus aureus Mycobacterium tuberculosis Pseudomonas aeruginosa Streptococcus pneumoniae

Streptococcus pneumoniae Explanation: Streptococcus pneumoniae (pneumococcus) is the most common cause of community-acquired pneumonia in people younger than 60 years without comorbidity and in those 60 years and older with comorbidity (Wunderink & Niederman, 2012). S. pneumoniae, a gram-positive organism that resides naturally in the upper respiratory tract, colonizes the upper respiratory tract and can cause disseminated invasive infections, pneumonia and other lower respiratory tract infections, and upper respiratory tract infections such as otitis media and rhinosinusitis. It may occur as a lobar or bronchopneumonic form in patients of any age and may follow a recent respiratory illness.

An x-ray of a trauma client reveals rib fractures and the client is diagnosed with a small flail chest injury. Which intervention should the nurse include in the client's plan of care? Suction the client's airway secretions. Immobilize the ribs with an abdominal binder. Prepare the client for surgery. Immediately sedate and intubate the client.

Suction the client's airway secretions. Explanation: As with rib fracture, treatment of flail chest is usually supportive. Management includes clearing secretions from the lungs, and controlling pain. If only a small segment of the chest is involved, it is important to clear the airway through positioning, coughing, deep breathing, and suctioning. Intubation is required for severe flail chest injuries, and surgery is required only in rare circumstances to stabilize the flail segment.

Which of the following is a clinical manifestation of a pneumothorax? Select all that apply. Sudden chest pain Asymmetry of chest movement Unilateral retractions Oxygen desaturation Bilaterally equal breath sounds

Sudden chest pain Asymmetry of chest movement Unilateral retractions Oxygen desaturation Explanation: Signs and symptoms of pneumothorax include sudden chest pain that is sharp and abrupt, a significant and sudden increase in shortness of breath, asymmetry of chest movement, unilateral retractions, bilateral differences in breath sounds, and/or oxygen desaturation. The patient with a pneumothorax would not have bilaterally equal breath sounds.

A client is prescribed postural drainage because secretions are accumulating in the upper lobes of the lungs. The nurse instructs the client to: Lay in bed with the head on a pillow. Take prescribed albuterol (Ventolin) before performing postural drainage. Perform drainage 1 hour after meals. Hold each position for 5 minutes.

Take prescribed albuterol (Ventolin) before performing postural drainage. Explanation: When a client is to perform postural drainage, the nurse should instruct the client to use the prescribed bronchodilator (eg, albuterol) first. This will open airways and promote drainage. The client is to perform postural drainage before meals, not after. This will aid in preventing nausea, vomiting, and aspiration. For secretions accumulated in the upper lobes, the client will sit up or even lean forward while sitting. Head on a pillow is not a sufficient increase in height. The client is also to lay in each position for 10 to 15 minutes.

For which reason does gas exchange decrease in older adults? The alveolar walls become thicker. The alveolar walls contain fewer capillaries. The elasticity of the lungs increases with age. The number of alveoli decreases with age.

The alveolar walls contain fewer capillaries. Explanation: Although the number of alveoli remains stable with age, the alveolar walls become thinner and contain fewer capillaries, resulting in decreased gas exchange. The lungs also lose elasticity and become stiffer. Elasticity of lungs does not increase with age, and the number of alveoli does not decrease with age.

Why would a client with COPD report feeling fatigued? Select all that apply. The client is using all expendable energy just to breathe. Muscle function gradually decreases over time in clients with COPD. The client is using all expendable energy for activities of daily living (ADLs). Lung function gradually decreases over time in clients with COPD.

The client is using all expendable energy just to breathe. Lung function gradually decreases over time in clients with COPD. Explanation: The client is using all expendable energy just to breathe. Lung function, not muscle function, gradually decreases over time in clients with COPD. In a client with COPD, fatigue and a feeling of exhaustion stem directly from the disease, not from activity level.

The nurse initiates the following intervention upon receiving a client back to the clinical unit after a throat-related procedure, "Elevate the head of the bed 45°." This assists in meeting which nursing goal? The client will have decreased pain. The client will remain alert and oriented. The client will have decreased edema. The client will have increased tissue perfusion.

The client will have decreased edema. Explanation: Elevating the head of the bed 45° when the client is fully awake decreases surgical edema and increases lung expansion. At this point in the recovery, elevating the head of the bed will not decrease the surgical pain as pain medication will be needed. Elevating the head of the bed will not affect mentation nor increase the blood supply.

A nurse observes a new environmental services employee enter the room of a client with severe acute respiratory syndrome. Which action by the employee requires immediate intervention by the nurse? The employee wears a gown, gloves, N95 respirator, and eye protection when entering the room. The employee doesn't remove the stethoscope, blood pressure cuff, and thermometer that are kept in the room. The employee removes all personal protective equipment and washes his hands before leaving the client's room. The employee enters the room wearing a gown, gloves, and a mask.

The employee enters the room wearing a gown, gloves, and a mask. Explanation: The nurse should tell the employee to wear the proper personal protective equipment, including a gown, gloves, N95 respirator, and eye protection, when entering the client's room. To prevent the spread of infection, a stethoscope, blood pressure cuff, and thermometer for single client use should be kept in the room of a client who requires isolation. Removing all personal protective equipment and washing hands before leaving the client's room are correct procedures.

Which homeostatic mechanism would the body of a critically ill client use to maintain normal pH? The lungs eliminate carbonic acid by blowing off more CO2. The lungs increase respiratory volume. The lungs retain more CO2 to lower the pH. The kidneys retain more HCO3 to raise the pH.

The lungs eliminate carbonic acid by blowing off more CO2. Explanation: To maintain normal pH in critically ill clients, the lungs eliminate carbonic acid by blowing off more CO2. To maintain normal pH in critically ill clients, the lungs conserve CO2 by slowing respiratory volume. The lungs would retain more CO2 during an acid-base imbalance in cases of metabolic alkalosis. The kidneys would retain more HCO3 to compensate during an acid-base imbalance in cases of metabolic acidosis.

A patient comes to the clinic complaining of a possible upper respiratory infection. What should the nurse inspect that would indicate that an upper respiratory infection may be present? The nasal mucosa The buccal mucosa The frontal sinuses The tracheal mucosa

The nasal mucosa Explanation: The nurse inspects the nasal mucosa for abnormal findings such as increased redness, swelling, exudate, and nasal polyps, which may develop in chronic rhinitis. The mucosa of the nasal turbinates may also be swollen (boggy) and pale bluish-gray. The nurse palpates the frontal and maxillary sinuses for tenderness, which suggests inflammation, and then inspects the throat by having the patient open the mouth wide and take a deep breath.

The nurse auscultated a patient's middle lobe of the lungs for abnormal breath sounds. To do this, the nurse placed the stethoscope on the: Posterior surface of the left side of the chest, near the sixth rib. Anterior surface of the right side of the chest, between the fourth and fifth rib. Posterior surface of the right side of the chest, near T3. Anterior surface of the left side of the chest, near the sixth rib.

The nurse auscultated a patient's middle lobe of the lungs for abnormal breath sounds. To do this, the nurse placed the stethoscope on the: Posterior surface of the left side of the chest, near the sixth rib. Anterior surface of the right side of the chest, between the fourth and fifth rib. Posterior surface of the right side of the chest, near T3. Anterior surface of the left side of the chest, near the sixth rib.

The nurse is instructing the patient with asthma in the use of a newly prescribed leukotriene receptor antagonist. What should the nurse be sure to include in the education? The patient should take the medication with meals since it may cause nausea. The patient should take the medication separately without other medications. The patient should take the medication an hour before meals or 2 hours after a meal. The patient should take the medication with a small amount of liquid.

The patient should take the medication an hour before meals or 2 hours after a meal. Explanation: The nurse should instruct the patient to take the leukotriene receptor antagonist at least 1 hour before meals or 2 hours after meals.

An client is described as having pectus carinatum. What would be the physical manifestation of this condition? The sternum protrudes and the ribs are sloped backward. The sternum is depressed from the second intercostal space. The thoracic and lumbar spine have a lateral S-shaped curvature. The chest is rounded, ribs are horizontal, and sternum is pulled forward.

The sternum protrudes and the ribs are sloped backward. Explanation: Also known as pigeon chest, in this congenital anomaly, the sternum abnormally protrudes and the ribs are sloped backward. A depressed sternum would be considered funnel chest, or pectus excavatum. S-shaped spinal curvature would be considered scoliosis. A rounded chest would be considered barrel chest in which the anteroposterior diameter increases to equal the transverse diameter.

While caring for a client with a chest tube, which nursing assessment would alert the nurse to a possible complication? Skin around tube is pink. Bloody drainage is seemed in the collection chamber. Absence of bloody drainage in the anterior/upper tube The tissues give a crackling sensation when palpated.

The tissues give a crackling sensation when palpated. Explanation: Subcutaneous emphysema is the result of air leaking between the subcutaneous layers. It is not a serious complication but is notable and reportable. Pink skin and blood in the collection chamber are normal findings. When two tubes are inserted, the posterior or lower tube drains fluid,whereas the anterior or upper tube is for air removal.

A nurse is caring for a client who has frequent upper respiratory infections. Which structure is most helpful in protecting against infection? Cilia Sinus cavity Tonsils Turbinates

Tonsils Explanation: Tonsils and adenoids do not contribute to respiration but protect against infection. Palatine tonsils are composed of lymphoid tissue. Cilia are fine hairs that move particles and liquid, preventing irritation and contamination of the airway. Sinuses are nasal cavity structures. Turbinates warm and add moisture to the inspired air.

A client is being mechanically ventilated in the ICU. The ventilator alarms begin to sound. The nurse should complete which action first? Notify the respiratory therapist. Manually ventilate the client. Troubleshoot to identify the malfunction. Reposition the endotracheal tube.

Troubleshoot to identify the malfunction. Explanation: The nurse should first immediately attempt to identify and correct the problem; if the problem cannot be identified and/or corrected, the client must be manually ventilated with an Ambu bag. The respiratory therapist may be notified, but this is not the first action by the nurse. The nurse should not reposition the endotracheal tube as a first response to an alarm.

Which technique does the nurse suggest to a client with pleurisy while teaching about splinting the chest wall? Turn onto the affected side. Use a prescribed analgesic. Avoid using a pillow while splinting. Use a heat or cold application.

Turn onto the affected side. Explanation: The nurse teaches the client to splint the chest wall by turning onto the affected side. The nurse also instructs the client to take analgesic medications as prescribed and to use heat or cold applications to manage pain with inspiration. The client can also splint the chest wall with a pillow when coughing. Reference:

A client with a decreased level of consciousness is in a recumbent position. How should the nurse best assess the lung fields for a client in this position? Inform that physician that the client is in a recumbent position and anticipate an order for a portable chest x-ray. Turn the client to enable assessment of all the patient's lung fields. Avoid turning the client, and assess the accessible breath sounds from the anterior chest wall. Obtain a pulse oximetry reading, and, if the reading is low, reposition the client and auscultate breath sounds.

Turn the client to enable assessment of all the patient's lung fields. Explanation: Assessment of the anterior and posterior lung fields is part of the nurse's routine evaluation. If the client is recumbent, it is essential to turn the client to assess all lung fields so that dependent areas can be assessed for breath sounds, including the presence of normal breath sounds and adventitious sounds. Failure to examine the dependent areas of the lungs can result in missing significant findings. This makes the other given options unacceptable.

Which intervention regarding nutrition is implemented for clients who have undergone laryngectomy? Use enteral feedings after the procedure Offer plenty of thin liquids when intake resumes Season food to suit an increased sense of taste and smell Recommend the long-term use of zinc lozenges

Use enteral feedings after the procedure Explanation: Enteral feedings are used 10 to 14 days after a laryngectomy to avoid irritation to the sutures and reduce the risk of aspiration. When oral intake resumes, the nurse offers small amounts of thick liquids. Following a laryngectomy, the client may experience anorexia related to a diminished sense of taste and smell. Excess zinc can impair the immune system and lower the levels of high-density lipoproteins ("good" cholesterol). Therefore, long-term or ongoing use of zinc lozenges to prevent a cold is not recommended. Reference:

For a client with advanced chronic obstructive pulmonary disease (COPD), which nursing action best promotes adequate gas exchange? Encouraging the client to drink three glasses of fluid daily Keeping the client in semi-Fowler's position Using a Venturi mask to deliver oxygen as ordered Administering a sedative as ordered

Using a Venturi mask to deliver oxygen as ordered Explanation: The client with COPD retains carbon dioxide, which inhibits stimulation of breathing by the medullary center in the brain. As a result, low oxygen levels in the blood stimulate respiration, and administering unspecified, unmonitored amounts of oxygen may depress ventilation. To promote adequate gas exchange, the nurse should use a Venturi mask to deliver a specified, controlled amount of oxygen consistently and accurately. Drinking three glasses of fluid daily wouldn't affect gas exchange or be sufficient to liquefy secretions, which are common in COPD. Clients with COPD and respiratory distress should be placed in high Fowler's position and shouldn't receive sedatives or other drugs that may further depress the respiratory center.

In general, chest drainage tubes are not used for a patient undergoing lobectomy. pneumonectomy. wedge resection. segmentectomy.

Usually no drains are used for a client undergoing pneumonectomy because the accumulation of fluid in the empty hemothorax prevents mediastinal shift. With lobectomy, two chest tubes are usually inserted for drainage, the upper tube for air and the lower tube for fluid. With wedge resection, the pleural cavity usually is drained because of the possibility of an air or blood leak. With segmentectomy, drains are usually used because of the possibility of an air or blood leak.

A mechanically ventilated client is receiving a combination of atracurium and the opioid analgesic morphine. The nurse monitors the client for which potential complication? Venous thromboemboli Pneumothorax Pulmonary hypertension Cor pulmonale

Venous thromboemboli Explanation: Neuromuscular blockers predispose the client to venous thromboemboli (VTE), muscle atrophy, foot drop, peptic ulcer disease, and skin breakdown. Nursing assessment is essential to minimize the complications related to neuromuscular blockade. The client may have discomfort or pain but be unable to communicate these sensations.

After lobectomy for lung cancer, a client receives a chest tube connected to a disposable chest drainage system. The nurse observes that the drainage system is functioning correctly when she notes tidal movements or fluctuations in which compartment of the system as the client breathes? Water-seal chamber Air-leak chamber Collection chamber Suction control chamber

Water-seal chamber Explanation: Fluctuations in the water-seal compartment are called tidal movements and indicate normal function of the system as the pressure in the tubing changes with the client's respirations. The air-leak meter — not chamber — detects air leaking from the pleural space. The collection chamber connects the chest tube from the client to the system. Drainage from the tube drains into and collects in a series of calibrated columns in this chamber. The suction control chamber provides the suction, which can be controlled to provide negative pressure to the chest.

A client is chronically short of breath and yet has normal lung ventilation, clear lungs, and an arterial oxygen saturation SaO2 of 96% or better. The client most likely has: poor peripheral perfusion. a possible hematologic problem. a psychosomatic disorder. left-sided heart failure.

a possible hematologic problem. Explanation: SaO2 is the degree to which hemoglobin (Hb) is saturated with oxygen. It doesn't indicate the client's overall Hb adequacy. Thus, an individual with a subnormal Hb level could have normal SaO2 and still be short of breath, indicating a possible hematologic problem. Poor peripheral perfusion would cause subnormal SaO2. There isn't enough data to assume that the client's problem is psychosomatic. If the problem were left-sided heart failure, the client would exhibit pulmonary crackles.

Another term for clergyman's sore throat is aphonia. chronic granular pharyngitis. atrophic pharyngitis. hypertrophic pharyngitis.

chronic granular pharyngitis. Explanation: In chronic granular pharyngitis, also referred to as clergyman's sore throat, the pharynx is characterized by numerous swollen lymph follicles. Aphonia refers to the inability to use one's voice. Atrophic pharyngitis is characterized by a membrane that is thin, white, glistening, and at times wrinkled. Hypertrophic pharyngitis is characterized by general thickening and congestion of the pharyngeal mucous membrane.

A nurse is assisting with a subclavian vein central line insertion when the client's oxygen saturation drops rapidly. He complains of shortness of breath and becomes tachypneic. The nurse suspects the client has developed a pneumothorax. Further assessment findings supporting the presence of a pneumothorax include: diminished or absent breath sounds on the affected side. paradoxical chest wall movement with respirations. tracheal deviation to the unaffected side. muffled or distant heart sounds.

diminished or absent breath sounds on the affected side. Explanation: In the case of a pneumothorax, auscultating for breath sounds will reveal absent or diminished breath sounds on the affected side. Paradoxical chest wall movements occur in flail chest conditions. Tracheal deviation occurs in a tension pneumothorax. Muffled or distant heart sounds occur in cardiac tamponade.

After undergoing a left thoracotomy, a client has a chest tube in place. When caring for this client, the nurse must: report fluctuations in the water-seal chamber. clamp the chest tube once every shift. encourage coughing and deep breathing. milk the chest tube every 2 hours.

encourage coughing and deep breathing. Explanation: When caring for a client who's recovering from a thoracotomy, the nurse should encourage coughing and deep breathing to prevent pneumonia. Fluctuations in the water-seal chamber are normal. Clamping the chest tube could cause a tension pneumothorax. Chest tube milking is controversial and should be done only to remove blood clots that obstruct the flow of drainage.

Class 1 with regard to TB indicates no exposure and no infection. exposure and no evidence of infection. latent infection with no disease. disease that is not clinically active.

exposure and no evidence of infection. Explanation: Class 1 is exposure but no evidence of infection. Class 0 is no exposure and no infection. Class 2 is a latent infection with no disease. Class 4 is disease, but not clinically active.

Which nursing diagnosis is most likely for a client who has just undergone a total laryngectomy? impaired verbal communication deficient knowledge risk for infection risk for chronic low self-esteem

impaired verbal communication

A client who has been diagnosed with an early glottis cancer would likely undergo which type of surgery? laser microsurgery vocal cord stripping partial laryngectomy total laryngectomy

laser microsurgery Explanation: In early glottis cancer, early stage lesions are treated and removed with a laser process. Vocal cord stripping would be the surgical treatment for early stage vocal cord lesions. Partial laryngectomy is done to treat early-stage laryngeal cancer when only one cord is involved. Total laryngectomy is done when the cancer extends beyond the vocal cords.

It is important for the nurse to provide required information and appropriate explanations of diagnostic procedures to clients with respiratory disorders in order to ensure adequate rest periods. manage respiratory distress. aid the client's caregivers. manage decreased energy levels.

manage decreased energy levels. Explanation: In addition to the nursing management of individual tests, clients with respiratory disorders require informative and appropriate explanations of any diagnostic procedures they will experience. Nurses must remember that for many of these clients, breathing may in some way be compromised and energy levels may be decreased. For that reason, explanations should be brief, yet complete, and may need to be repeated later after a rest period. The nurse must also ensure adequate rest periods before and after the procedures. After invasive procedures, the nurse must carefully assess for signs of respiratory distress.

The nurse is caring for a client following a tonsillectomy and adenoidectomy. Two hours after the procedure, the client begins to vomit large amounts of dark blood at frequent intervals and is tachycardic and febrile. After notifying the surgeon, the nurse stays with and closely monitors the client. obtains a light, mirror, gauze, and curved hemostats. prepares for a needle aspiration. orally suctions the client, as needed.

obtains a light, mirror, gauze, and curved hemostats. Explanation: If the client vomits large amounts of dark blood at frequent intervals, if the pulse rate and temperature rise, or if the client becomes restless, the nurse notifies the surgeon immediately. The nurse should have the following items ready for examination of the surgical site for bleeding: a light, a mirror, gauze, curved hemostats, and a waste basin. It is not necessary for the nurse to stay at the client's bedside. Needle aspiration is a procedure considered for clients experiencing a peritonsillar abscess. Although oral suctioning may be needed at some point of care, it is not a priority at this time.

A client who underwent thoracic surgery to remove a lung tumor had a chest tube placed anteriorly. The surgical team places this catheter to: remove air from the pleural space. remove fluid from the lungs. administer IV medication. ventilate the client.

remove air from the pleural space. Explanation: After thoracic surgery, draining secretions, air, and blood from the thoracic cavity is necessary to allow the lungs to expand. A catheter placed in the pleural space provides a drainage route through a closed or underwater-seal drainage system to remove air. Sometimes two chest catheters are placed following thoracic surgery: one anteriorly and one posteriorly. The anterior catheter removes air; the posterior catheter removes fluid.

Influenza, an annual epidemic in the U.S., creates a significant increase in hospitalizations and an rise in the death rates from pneumonia and cardiovascular disease. Besides death, what is the most serious complication of influenza? staphylococcal pneumonia tracheobronchitis cardiovascular disease viral pneumonia

staphylococcal pneumonia Explanation: Complications include tracheobronchitis, bacterial pneumonia, and cardiovascular disease, however staphylococcal pneumonia is the most serious complication.

In relation to the structure of the larynx, the cricoid cartilage is the only complete cartilaginous ring in the larynx. used with the thyroid cartilage in vocal cord movement. the largest of the cartilage structures. the valve flap of cartilage that covers the opening to the larynx during swallowing.

the only complete cartilaginous ring in the larynx. Explanation: The cricoid cartilage is located below the thyroid cartilage. The arytenoid cartilages are used with the thyroid cartilage in vocal cord movement. The thyroid cartilage is the largest of the cartilage structures and part of it forms the Adam's apple. The epiglottis is the valve flap of cartilage that covers the opening to the larynx during swallowing.

A nurse is caring for a client who has a tracheostomy tube and who is undergoing mechanical ventilation. The nurse can help prevent tracheal dilation, a complication of tracheostomy tube placement, by: suctioning the tracheostomy tube frequently. using a cuffed tracheostomy tube. using the minimal-leak technique with cuff pressure less than 25 cm H2O. keeping the tracheostomy tube plugged.

using the minimal-leak technique with cuff pressure less than 25 cm H2O. Explanation: To prevent tracheal dilation, a minimal-leak technique should be used and the pressure should be kept at less than 25 cm H2O. Suctioning is vital but won't prevent tracheal dilation. Use of a cuffed tube alone won't prevent tracheal dilation. The tracheostomy shouldn't be plugged to prevent tracheal dilation. This technique is used when weaning the client from tracheal support.


Kaugnay na mga set ng pag-aaral

Quiz 12 : Essentials of Networking

View Set

Primerica Session A Chapter 2 Contract Law (Chapter Quiz, Snap Shots, and review questions)

View Set

Sensation and Perception Module 18

View Set

Computer Technology I Glossary 4: Software

View Set

OMIS 360 EXAM 1 Disc Ques Dr. Huynh

View Set

MGMT 4860 - Organizational Design and Change - Midterm Study Guide

View Set

Global Dimensions Bus 187 SB CH20

View Set

Developmental Psychology Chapter 6

View Set